7116273 aptitude igate infosys tcs ect

126
iGate test conducted on 16.10.2004. at Crescent engineering college Aptitude 1. Select the odd one (a) January (b) February (c) Wednesday (d) November 2. Select the antonym of capture from the following (a) attack (b) Release (c) condemn (d) None of the above 3. Find the antonym of autumn (a) Spring (b) Winter (c) Summer (d) None of the above 4. One skirt requires 3.75 yards of cloth. How many skirts you can make from 45 yards? Ans: 12 skirts 5. Last month of an year (a) January (b) February (c) December (d) November 6. Is the meaning of Client and Customer, (a) same (b) contradictory (c) no relation 7. Is the meaning of Canvas and Canvass, (a) same (b) contradictory (c) no relation 8. Is the meaning of Ingenious and Ingenuous, (a) same (b) contradictory (c) no relation 9. Is the meaning of Credible and Credulous, (a) same (b) contradictory (c) no relation 10. Select the odd one out. (a) 1/4 (b) 1/3 (c) 1/6 (d) 1/18 11. Select the least from the following. (a) 0.99 (b) 1 (c) 81 (d) 0.333 12. Find the next number in the series. 1, 0.5, 0.25, 0.125 Ans: 0.0625 13. One do llar is saved in one month. Then how much dollar is saved in one day? Ans: 1/30 =0.0333$ 14. Y catches 5 times more fishes than X. If total number of fishes caught by X and Y is 48, then number of fishes caught by X? Ans: 8 15. If a train covers 600m in 0.5 seconds, how long it will cover in 10 seconds? Ans: 3000m = 3km 16. The girl's age is twice that of boy, if the boy is four years old. After four years the age of the girl is Ans: 12 years 17. Sister's age is twice than that of the brother. If the brother's age is six, what is the sister's age after two years? Ans: 14 Yrs. 18. Two lemons cost 10 cents. Then one and a half dozen cost Ans: 90 cents 19. A clock is late by 1 minute 27 seconds in a month. Then how much will it be late in 1 day? Ans: 2.9 seconds 20. Opposite of Remote? (a) Far (b) Near (c) Huge (d) Village 21. Statement A: All great men are ridiculous; Statement B: I am ridiculous ; Inference : I am a great man; (a) True (b) False (c) Not clear

Upload: dinesh-sugumaran

Post on 18-Nov-2014

253 views

Category:

Documents


1 download

TRANSCRIPT

Page 1: 7116273 Aptitude IGATE Infosys Tcs Ect

iGate test conducted  on 16.10.2004. at  Crescent engineering college

Aptitude

1. Select the odd one  (a) January (b) February (c) Wednesday (d) November 

2. Select the antonym of capture from the following 

(a) attack (b) Release (c) condemn (d) None of the above 

3. Find the antonym of autumn (a) Spring (b) Winter (c) Summer (d) None of the above

4. One skirt requires 3.75 yards of cloth. How many skirts you can make from 45 yards?  Ans: 12 skirts

5. Last month of an year  (a) January (b) February (c) December (d) November 

6. Is the meaning of Client and Customer,  (a) same (b) contradictory (c) no relation 

7. Is the meaning of Canvas and Canvass,  (a) same (b) contradictory (c) no relation 

8. Is the meaning of Ingenious and Ingenuous,  (a) same (b) contradictory (c) no relation 

9. Is the meaning of Credible and Credulous,  (a) same (b) contradictory (c) no relation 

10. Select the odd one out.  (a) 1/4 (b) 1/3 (c) 1/6 (d) 1/18 

11. Select the least from the following.  (a) 0.99 (b) 1 (c) 81 (d) 0.333 

12. Find the next number in the series. 1, 0.5, 0.25, 0.125  Ans: 0.0625 

13. One do llar is saved in one month. Then how much dollar is saved in one day?  Ans: 1/30 =0.0333$ 

14. Y catches 5 times more fishes than X. If total number of fishes caught by X and Y is 48, then number of

fishes caught by X?  Ans: 8 

15. If a train covers 600m in 0.5 seconds, how long it will cover in 10 seconds? Ans: 3000m = 3km 

16. The girl's age is twice that of boy, if the boy is four years old. After four years the age of the girl is 

Ans: 12 years 

17. Sister's age is twice than that of the brother. If the brother's age is six, what is the sister's age after two

years? Ans: 14 Yrs. 

18. Two lemons cost 10 cents. Then one and a half dozen cost  Ans: 90 cents 

19. A clock is late by 1 minute 27 seconds in a month. Then how much will it be late in 1 day? 

Ans: 2.9 seconds 

20. Opposite of Remote? (a) Far (b) Near (c) Huge (d) Village 

21. Statement A: All great men are ridiculous;  Statement B: I am ridiculous ; Inference : I am a great man; 

(a) True (b) False (c) Not clear 

22. Statement: Normal children are active; Inference: All children are active; 

(a) True (b) False (c) Uncertain 

23. Next number in the series 1, 1/2, 1/4, 1/8 ?  Ans: 1/16 

24. In 6 seconds a light flashes once. In one hour how many times it will flash?  Ans: 601 times 

25. At 20% discount, a cycle is sold at a selling price of 2500 Rs. What is the actual price? Ans: Rs. 3125 

26. Statement A: A & B have same age; Statement B: B is younger than C; Inference : A is younger than C; 

(a) True (b) False (c) Uncertain 

27. A invests $12000, B invests $8000, C invests $6000 and they got a profit of $1200. How much share A

got more than B and C?  Ans: 2/13 and 3/13 

 Technical   Questions contain Electronics and Computer Science

1. Full form of TTL and CMOS 

2. Which is a good conductor (Extrinsic or Intrinsic) 

3. What are the different types of capacitors (Electrolytic,  dielectric...etc) 

4. Select a passive component from the following (four choices were  there) 

Page 2: 7116273 Aptitude IGATE Infosys Tcs Ect

5. Minimum no. of lines required for communication using RS232

6. To convert 1's complement to 2's complement and vice versa 

7. During which time we use 'size of' command.

8. Out of four choice we have to identify which is a macro. 

9. There was one pointer _expression related question. 

10. To find post fix _expression. 

11. What type of operating system is unix    a)pre-emptive b)non-preemptive c)batch 

12. Where we use DFD(Data flow design)

   a)structural languages b)object oriented languages c)UML d)none of these

13. Name the error which occurs when we write on a page

  a) segment fault b)permission fault c) page fault 

14. A question based on the representation of an array in C  An array whose elements are fn pointers which

inturn returns a character 

TCS ENGLISH  SAMPLE PAPER 23rd SEPTEMBER 2006 AT HYDERABAD

Word Meaning Synonyms Antonyms Abysmal Adj. Bottomless Use : His arrogance is

exceeded only by his abysmal ignorance

Terrible, awful, dreadful, appalling, very bad,

Superb

acronym n. A word formed by the initial letters of a multi-word name

Contraction, ellipsis,  

admonish v. warn strongly, reprove Use: He admonished his listeners to change their wicked ways

Reprove, reprimand, chide, rebuke, usurp

Approve

Circumspect v. Investigation before acting,

Use: She tried always to be circumspect

Cutious, prudent, careful, guarded, wary, judicious, vigilant,

reckless

Conciliation n. the act of placating Appeasement, pacification, propitiation

Incitement

Connotation n. Suggested or implied meaning of an expression. Use: Foreigners frequently are unaware of the connotations of the words they use.

Nuance, suggestion, implication, undertone, overtone, subtext,

 

covert Adj. secret or hidden, not openly practiced or vowed n. a covering that serves to conceal or shelter something Use: Investigations of CIA reveal that such covert operations can get out of control

Clandestine, concealed, stealthy, underground, copse, wood, thicket

Open

Covetous Adj. avaricious, eagerly desirous of Use: The child was covetous by nature and wanted to take the toys belonging to his classmates

envious, jealous, desirous, greedy

Generous, temperate

Deprecate v. express disapproval of, protest against, belittle Use: A firm believer of old-fashioned courtesy, Miss Post, deprecated the modern tendency to address new acquaintances by their first names.

Denounce, deplore, condemn, censure, denigrate, decry, belittle.

Approve

Page 3: 7116273 Aptitude IGATE Infosys Tcs Ect

Diligence n. care and perseverance in carrying out tasks. Use : Her employers were greatly impressed by her diligence and offered her a partnership in the firm

Hard-working, assiduousness, meticulousness, conscientiousness, painstakingness

Laziness, carelessness,

negligence

Discretion n. ability to adjust action to circumstances Use: Use your discretion in this matter and do not discuss the matter with anyone.

Prudence, caution, acumen, predilection, sagacity, wariness, volition

Indiscretion

dispel v. scatter, drive away, cause to vanish Use: The bright sunlight eventually dispelled the morning mist

Disperse Attract

Dissemination v. Opening to public discussion or debate, the act of spreading something, Use:By their use of the internet, propagandists, have been able to disseminate their pet doctrines to new audiences around the globe

Broadcasting, diffusion, propagation,

 

Dogmatic Adj. Opinionated Use: We tried to discourage him from being so dogmatic, but nothing could convince him that his opinions might be wrong

Arbitrary, doctrinal, unbending, inflexible, authoritarian,

Flexible

echelon n. A body of troops arranged in a line Level, stratum, rank, height

 

fallible Adj. Liable to err. Use: I Know I am fallible, but I feel confident that I am right this time

Imperfect, unsound, mortal, weak, frail,

Perfect

Harbinger n. an indication of the approach of something or someone, forerunner v. fore shadow or presage Use: The crocus is an early harbinger of spring

Forerunner, herald, potent, omen,

 

illustrious Adj. Widely known and esteemed Memorable, well-known, famous

Inglorious, shameful

impetus n. A force that moves something along, incentive, stimulus Use: A new federal highway program would create jobs and will give added impetus to our economic recovery

Momentum, thrust, motivation

inertia

Intermittent Adj. Periodic Use: The outdoor wedding reception had to be shifted indoor to avoid the intermittent showers that fell on and of all afternoon.

Alternating, sporadic Constant

Latent Adj. Potentially existing, but not presently evident or realized Use: Polaroid pictures are popular at the parties because you can see the latent photographic image gradually appear before your eyes

Dormant, embryonic, suppressed, undeveloped,

Overt

Latitude n. freedom from narrow limitations Use: I think you have permitted your son too much latitude in this manner

Leeway, freedom, autonomy, liberty, room, rope

 

Manifestation n. outward demonstration, indication Use: Mozart’s early attraction to the harpsichord was the first manifestation of his pronounced musical bent

Sign, demonstration, expression, symptom,

 

Page 4: 7116273 Aptitude IGATE Infosys Tcs Ect

mitigate v. lessen or try to lessen the seriousness or extent of Use: Nothing he could do to mitigate her anger, she refused to forgive him

Appease, alleviate, allay, assuage, mollify, extenuate, palliate

Aggrevate, exacerbate

motley Adj. Multicolored or mixed Use: he wore a motley tunic, red and green and blue and gold all patched together haphazardly

Assorted, diversed, mixed, dissimilar,

Uniform, homogenous, monochrome

pensive Adj. Dreamily thoughtful, thoughtful with a hint of sadness Use: The pensive lover gazed at the portrait of his beloved and sighed deeply.

Brooding, pondering, meditative, contemplative

 

prolific Adj. Abundantly fruitful Use: She was a prolific writer, who produced as meany as three books a years

Productive, abundant, fecund

Unproductive, scarce

relinquish v. give up something with reluctance, yield Use: once you get used to fringes like expense account meal and company car, it is very difficult to relinquish them

Surrender, renounce, abandon, repudiate, cede,

 

Vanquish, retain,

solicit v. request earnestly, seek Use: The Mayor telephoned all the member of the city council to solicit their votes

Importune, implore, crave, beseech,

Grant

tractable Adj. Docile, easily managed, susceptible to suggestion Use: Although Susan seemed to be a tractable young woman, she had a stubborn streak of independence

Obedient, dutiful, well-mannered

Disobedient, intractable

veer v. change in direction Use: After what seemed to be en eternity, the wind veered to the east and the storm abated

Turn, swerve  

Vehement Adj. Forceful, intensely emotional; with marked vigor Use: He became so Vehement in describing what had happened with him that he started jumping up and down

Fervent, passionate, Apathetic

Caprice n. whim, a sudden desire. Use: She was an unpredictable creature, acting on caprice, never taking thought of consequences

Whim, impulse, quirk, fad, fancy, notion

 

stifle v. suppress, extinguish, inhibit Use: Halfway through the boring picture, Laura gave up trying to stifle he yawns.

Smother, asphyxiate, choke, suffocate, strangle, curb, restrain, repress,

Let out

concur v. agree Use: Did you concur with the decision of the court or did you find it unfair?

Agre, harmonize, coincide, assent, acquiesce

Conflict, resist

lethargy v. lack of vitality or energy Use: A complete days work left him in a state of lethargy

Stupor, indolence, weariness

Get-up-and-go

Furtive Adj. stealthy, sneaky, secret and sly or sordid Use: noticing the furtive glance the customer gave the diamond bracelet on the counter, the jeweler wondered whether he had a potential shoplifter in his hands

Surreptitious, clandestine

open

Page 5: 7116273 Aptitude IGATE Infosys Tcs Ect

efface v. wipe out or make dim Use: The coin had been handled so many times that its date had been effaced

Obliterate, eradicate, Engrave

Pretentious Adj. Intended to attract notice and impress others, making unjustified claims, overambitious Use: The other prize winner isn’t wearing her medal.; isn’t it a bit pretentious of you to wear yours?

Ostentatious, pompous, conceited

Down-to-earth

compunction n. a feeling of deep regret (usually of some misdeed) Use: The judge was especially sever in his sentencing as the criminal had shown no compunction for his heinous crime

Regret, scruple, qualm, guilt, reluctance, hesitation,

 

confiscate      

Infosys

INFOSYS PLACEMENT PAPER ON 10 th SEPTEMBER2006

Exercise 1: The passage given below is followed by questions based on its content. Read the passage & choose the best answer 4 the questions

  The Death Car 

It was cold night in September. The rain was drumming on the car roof as George & Marie Winstion drove through the empty country roads towards the house of their friends, the Harrissons, where they were going to attend a party to celebrate the engagement of the Harrisons daughter, Lisa. As they drove, they listened to the local radio station, which was playing classical music. They were about 5 miles from the destination when the music on the radio was interrupted by a news announcement: “The Cheshire police have issued a serious warning after a man escaped from Colford Mental Hospital earlier this evening. The man, John Downey, is murderer who killed 6 people before he was captured 2 years ago. He is described as large, very strong & extremely dangerous. People in the Cheshire area are warned to keep their doors & windows locked, & to call the police immediately if they se anyone acting strangely.” Marie shivered, “A crazy killer. And he’s out there somewhere. That’s scary.”“Don’t worry about it,” said her husband. “We’re nearly there now. Anyway, we have more important things to worry about. This car is losing power for some reason—it must be that old problem with the carburetor, If it gets any worse, we’ll have to stay at the Harrisons’ tonight & get it fixed before we travel back tomorrow,” As he spoke, the car began to slow down, George pressed the accelerator, but the engine only coughed. Finally they rolled to a halt, as the engine died completely, Just as they stopped, George pulled the car off the road, & it came to rest under a large tree. “Blast!” said George angrily. “Now we’ll have to walk in the rain.” “But that’ll take us an hour at least,” said Marie. “And I have my high-held shoes & my nice clothes on. They’ll be ruined!” “Well, you’ll have to wait while I run to the nearest house & call the Harissons. Someone can come out & picks us up,” said George. “ButGeorge! Have you forgotten what the radio said? There’s a homicidal maniac out there! You can’t leave me alone here!” “You’ll have to hide in the back of the car. Lock all the doors & lie on the floor in the back, under this blanket. No-one will see you, when I come back, I’ll knock 3 times on the door. Then you can get up & open it. Don’t open it unless you here 3 knocks.” George opened the door & slipped out into the rain. He quickly disappeared into the blackness. Marie quickly locked the doors & settled down under the blanket in the back for a long wait. She was frightened & worried, but she was a strong-minded woman. She had not been waiting long, however, when she heard a strange scratching noise. It seemed to be coming from the roof of the car. Marie was terrified. She listened, holding her breath. Then she heard 3 slow knocks, one after the other, also on the roof of the car. Was it her husband? Should she open the door? Then she heard another knock, and another. This was not her husband.

Page 6: 7116273 Aptitude IGATE Infosys Tcs Ect

It was somebody--or something--else. She was shaking with fear. But she forced herself to lie still. The knocking continued-- bump, bump, bump, bump Many hours later, as the sun rose, she was still lying there. She had not slept for a moment. The knocking had never stopped, all night long. She did not know what to do. Where was George? Why had he not come for her? 

Suddenly, she heard the sound of 3 or 4 vehicles, racing quickly down the road. All of them pulled up around her, their tires screeching on the road. At last! Some one had come! Marie sat up quickly & looked out the window. 

The 3 vehicles were all police cars, & 2 still had their lights flashing. Several policemen leap out. One of them rushed towards the car as Marie opened the door. He took her by the hand. 

“Get out of the car & walk with me to the police vehicle. miss. You’re safe now. Look straight ahead. Keep looking at police car. Don’t look back. Just don’t look back.” 

Something in the way he spoke filled Marie with cold horror. She could not help herself. After 10 yards from the police car, she stopped, turned & looked back at the empty vehicle. 

George was hanging from the tree above the car, a rope tied around his neck. As the wind blew his body back & forth, his feet were bumping gently on the roof of the car-- bump, bump, bump, bump 

1) What was the reason for the news announcement on the radio?a) 6 people. Including John Downey, had been murdered?b) A dangerous prisoner had escapedc) The police were warning of accidents on the roads in the bad weatherd) Some people had bens en acting strangely in the Cheshire area

  2) What did George think was causing trouble with the car?

a) The carburetor b) The rain drumming on the roofc) The accelerator d) He had no idea

  3) Why did he pull the car off the road?

a) To have a rest b) To go for a walkc) To walk to the nearest house d) It broke down

  4) Why did Marie stay in the car when George left?

a) She was afraid to go out in the dark b) So no one could steel the carc) Her clothes weren’t suitable for the rain d) She wanted to get some sleep

  5) Where did George set off to walk?

a) The mental hospital b) The nearest housec) The Harrison’s house d) The police station

  6) What made Marie so frightened as she waited in the car?

a) There was a strange sound coming from the roofb) She could see a man strangely outside the carc) Some police cars came racing down the roadd) She was afraid of the rain and the dark

Exercise 2: Each sentence below has 1 or 2 blanks – each blank indicating that something has been omitted. Beneath the sentence are some words. Choose the word for each blank that best fits the meaning of the sentence as a whole

Page 7: 7116273 Aptitude IGATE Infosys Tcs Ect

7) Athletes have so perfected their techniques in track and field events that the _________ becomes _________ before record books

a) Announcement …………public b) Meet…………………….officialc) Time…………………….authentic d) Fantastic………………...common place

  8) A________ child, she was soon bored in class; she already knew more mathematics than her junior school teachers

a) Obdurate b) Precocious c) Recalcitrant d) Contemporary  9) The subtle shades of meaning, & still subtler echoes of association, make language an instrument which scarcely anything short of genius can wield with __________ & __________

a) Confidence----------aloofness b) Definiteness---------certaintyc) Sincerity--------------hope d) Eloquence------------ruthlessness

  10) Unwilling to admit that they had been in error, the researchers tried to_______ tried case with more data obtained from dubious sources

a) Ascertain b) Buttress c) Refute d) Dispute  11) His one vice was gluttony & so it is not surprising that as he aged he became increasingly____________

a) Despondent b) Corpulent c) Carping d) Lithe

 Exercise 3: Please read all the questions in the table below (12-21) as one continuous passage. Tick the varb with right tense or the correct word to fill in the gaps in each of the sentences.

12) A famous singer had been contracted to sign at a Paris opera house & ticket sales _________ booming.a) is b) are c) were d) have been13) In fact, the night of the concert, the house was packed; every ticket ________________a) is selling b) was selling c) sold d) had been sold14) The feeling of anticipation & excitement was in the air as the house manager__________ the stage & said, “Ladies & gentlemen, thank you for your enthusiastic support!a) took b) takes c) had taken d) was taking15) I am afraid that due to illness, the man whom you’ve all come to hear________________ performing tonighta) will not be b) has not been c) had not been d) was not16) However, we _________ a suitable substitute who, we hope, will provide you with comparable entertainment.”a) are finding b) were finding c) had found d) have found17) The crowd____________ in disappointment & failed to hear the announcer mention the stand-in’s namea) groans b) groaned c) had groaned d) were groaning18) The environment turned from excitement to frustration . The stand-in performer__________ the performance everything he had.a) will give b) had given c) gave d) gives19) When he had finished, there was nothing but an uncomfortable silence. No one ___________a) Applauded b) Applauds c) Was applauding d) Has applauded20) Suddenly, from the balcony, a little boy stood up and____________, “Daddy, I think you’re wonderful!”a) Applauded b) Applauds c) Was applauding d) Has applauded21) The crowd_________________ into thunderous applausea) breaks b) broke c) had broken d) was breaking

Page 8: 7116273 Aptitude IGATE Infosys Tcs Ect

Exercise 4: From each group of sentences given below, indicate the sentence that contains the error:

  22) Group 1 a) Driving long distances causes sleepiness, & sleepiness causes serious accidents.b) On a table at the rear of the room was a notebook, a pair of scissors, & a biology textbookc) Finally, there seems to be a growing interest in vegetarianism in this countryd) Either the local chief of police or his officers are guilty of violating the rights of prisoners

  23) Group 2

a) Simple cookbooks for inexperienced cooks have become quite popular in recent years they are available at many bookstores

b) Some cookbooks, such as The Joy of cooking, have been classics for generationsc) One popular cookbook is The Art of French Cooking, by Julia Child, a colorful character

who charmed television audiences for many yearsd) The Art of French Cooking blends classic recipes with meticulous explanation; ordinary

cooks find the recipes manageable  24) Group 3

a) Around 50% of the forest are destroyed every yearb) The bus leaves tomorrow morningc) A tiger is a dangerous animald) Can you please the sugar?

  25) Group 4

a) There must be some mistake. I should have scored more marksb) The number of trainees are hundredc) 50% of the houses need repairsd) The Commissioner, along with his family members was seen the party

  26) Group 5

a) The scissors is very sharp b) Congratulations are in orderc) One of the cases is open d) She plays tennis well but she’ll never be a Steffi Graf

 

Exercise 5: Please mark the correct statement from the pairs given below:

  27) Pair 1 a) Repeated occurrences cannot be ignored b) Repeated occurences cannot be ignored

  28) Pair 2

a) We need to get a consensus on the decisionb) We need to get a concensus on the decision

  29) Pair 3

a) Only authority personnel are allowed in this areab) Only authorized personnel are allowed in this area

   30) Pair 4

a) The actress decided to sue the sleazy tabloid for deformation of her characterb) The actress decided to sue the sleazy tabloid for defamation of her character

  31) Pair 5

a) Everyone knows that Hogwarts in the Harry Potter series is a mythical schoolb) Everyone knows that Hogwarts in the Harry Potter series is a legendary school

 

Page 9: 7116273 Aptitude IGATE Infosys Tcs Ect

32) Pair 6 a) Most people think caffeine is not good for healthb) Most people think caffiene is not good for health

Exercise 6: Select the best word/phrase/line to complete each sentence in the most appropriate manner

33) ‘Reema’s bad-mouthing Peter only because she is jealous of him.’ Means______________a) Peter really is a nice person b) Peter really is a mean personc) Peter really is a difficult person d) Peter really is a tough person

  34) If some one is “gung ho”, they are_______

a) stupid b) Childish c) Enthusiastic d) Loud  35) Mr. Hughes has been asked to___________ this difficult project because of his experience working for many years in Iran

a) undergo b) understand c) undervalue d) undertake  36) ‘Stop talking to those angry men, you are just adding fuel to the fire’ is the same as________

a) Stop talking to those angry men, you are just coming in the wayb) Stop talking to those angry men, you are just making it worsec) Stop talking to those angry men, you are just adding to the noised) Stop talking to those angry men, you are just talking too much

  37) ‘Sudhir’s work is behind schedule – I think he bit more than he could chew’ is the same as________

a) Sudhir has taken too much of work b) Sudhir takes very long breaksc) Sudhir does not know how to do the work d) Sudhir is a lazy person

  38) There are many__________ to our rules, and I do not think that’s fair.

a) examples b) exceptions c) instances d) provisions

Exercise 7: Choose the correct / most appropriate word/s to fill in the gap in the sentences given below.  

39) I didn’t set _________ to do this but I’m pleased with the result.a) in b) out c) on d) down

  40) This looks too heavy,______________ pick it up?

a) Can I b) may I c) need I d) would I  41) I am glad so many people have passed the test. In fact, there were_________ who haven’t.

a) little b) a little c) few d) a few  42) Pope John Paul II ___________ more than 90 countries.

a) has visited b) was visited c) visits d) has been visiting  43) I _____________ Carl since I ______________ a little child.

a) have known, have been b) have known, wasc) knew, have been d) knew, was

  44) I wonder if _____________ will show up at the meeting?

a) someone b) anyone c) one d) everyone  45) Have you given up______________.

a) to smoke b) smoke c) some smoking d) smoking

Page 10: 7116273 Aptitude IGATE Infosys Tcs Ect

Aptitude General :

DRDO SAMPLE INTERVIEW QUESTIONS

1) Coldest planet:Pluto

2) INS Shivali is the first:

3) Which one of the following was NOT indegineously developed?:Prithvi/Akash/Agni

4) Full form of SARS

5) Anthrax is a :Virus/Bacteria/.../...

6) Dakshina Gangothri is:Ganga's origin/Indian camp @ antartica/.../...

7) Which of the following is a chemical weapon:Mustard Gas/Marsh Gas/.../...

8) A question based on Coding and Decoding

9) Another question similar to above

10) Question on series completion

11) Another series completion question

12) Where is Institute of Forensic Science?:Hyderabad

13) A G.K question based on X and Y chromosomes in males and females

                                                                                                  

Sample technical questions asked in test last year in CSE :

1) Banker's algorithm is used for: Deadlock Avoidance

2) A LOT of questions were based on generating strings from a given grammar. 

3) A circle(dot) shown in the PCB is:Vcc/Grnd/Pin 1/Pin 14

4) Program Segment Prefix in MS-DOS 5.0 is:

5) Some IP addresses were given and the question was to select the private addess from it(?)

6) 10Base2 and 10Base5 wires refers to:

7) A question on sliding-window protocol

8) Which of the following require a driver?:disk/cache/ram/cpu

9) A LOT of mathematical questions which were asked from calculus,trigonometry...

The questions asked in ECE were mainly from Control Systems, Communications

EMT and microprocessor

Make sure that u know the fundas of microprocessors useful in interview also: see if u know these

questions

1. Which type of architecture 8085 has? 

2. How many memory locations can be addressed by a microprocessor with 14 address lines? 

3. 8085 is how many bit microprocessor? 

4. Why is data bus bi-directional? 

5. What is the function of accumulator? 

6. What is flag, bus? 

7. What are tri-state devices and why they are essential in a bus oriented system? 

8. Why are program counter and stack pointer 16-bit registers? 

9. What does it mean by embedded system? 

10. What are the different addressing modes in 8085? 

11.What is the difference between MOV and MVI? 

12. What are the functions of RIM, SIM, IN? 

13. What is the immediate addressing mode? 

14. What are the different flags in 8085? 

15. What happens during DMA transfer? 

16. What do you mean by wait state? What is its need? 

17. What is PSW? 

Page 11: 7116273 Aptitude IGATE Infosys Tcs Ect

18. What is ALE? Explain the functions of ALE in 8085. 

19. What is a program counter? What is its use? 

20. What is an interrupt? 

21. Which line will be activated when an output device require attention from CPU? 

                                                                                                   

Then comes the interview questions asked in ECE interview were fundamental. Qustions asked in my

interview were:

Director

1. Which college and university are you coming from?

2. Did you appear for GATE? Why are you not interested in higher studies?

3. Did you appear for IES?

4. Did you appear for any other board interview of public sector?

5. The subjects you have learned in college can be divided into three- basic electronics, communi-cation

and digital logic. Tell me any five subjects you like.

(I told radar and navigational aids, electronic warfare, satellite communication, biomedical

instrumentation, fuzzy electronics and basic digital electronics as my subjects)

Board member1 (QUESTION LEVEL- MODERATE)                

1. Write the truth table for full adder and implement it in NAND gate only.

2. What's the difference between looping 0s and 1s in K map?

3. Difference between microprocessor and micro controller

4. Microprocessors you are familiar with

5. How will you send and receive data to a micro-processor? (One method is I/O mapped I/O which is the

other one?)

6. Radar range equation?

7. Does the radar range depend upon the frequency of the signal transmitted?

8. What is Doppler shift? What is its importance?

BOARD MEMBER -2 QUESTION LEVEL- TOUGH) 

1. I will make two fuzzy statements. Pencil is long. Table is long. What is the term long signify?

2. What is a membership function?

3. What are the design criteria for very low frequency amplifier?

4. Can you measure distance with the help of CW radar? If so how?

5. How will you design a stable oscillator? (Not with crystal oscillator because temperature affects it)

6. You have designed an amplifier. After few days it is found that its gain have changed. What might be

the reason?                          

                                                                                                   

BOARD MEMBER-3 (QUESTION LEVEL- MODERATE) 

1. A plane is moving in a circular path around the transmitter of the radar. Will there be Doppler shift

detected in the radar?

2. State Keplers laws

3. Why there is more geo synchronous satellite?

4. The angular difference between two satellites is 2 degree. What is the maximum number of satellites

needed to cover the whole earth?

5. What is the minimum number of satellites needed to cover the whole earth?

BOARD MEMBER-4 QUESTION LEVEL- MODERATE) 

Page 12: 7116273 Aptitude IGATE Infosys Tcs Ect

1. Which is the law of conservation involved in the second of Keplers?

2. Why do you explain elliptical orbit while stating Kepler's law? Why not circular orbit?

3. What are the advantages of optical communication?

4. What are the invasive and non-invasive methods of instrumentation?

For CS guys they started with this question: What is a key board? Where u will connec? What will happen

if you press the keys?..

For maths guys they asked some questions on series.. I don't know muchSome guys were selected just by

describing the final year project.    

                                                                                                  

1. How can you design a phase detector using a XOR gate? 

2. Questions abt differentiator and integrator. What will happen if we increase/decrease the values of

R/C? 

3. how will a low/high pass filters behave to different signals –ramp, pulse etc 

4. questions on flip flops 

5. Johnson counter 

6. Questions on microprocessors- what is SIM? 

7. Abt your project. What will happen when this/that happens to your project? 

8. Radar, antenna and satellite communication. 

9. Which is the first/latest communication satellite? 

10. What is apogee /perigee?

TVS LUCAS PAPER

The test was for 1 hour containing 100 questions. 50 and above was the cut off for selection. Negative marking was there. Easy questions will be at the end. So don’t waste time answering difficult questions. Quants was dead easy.(no preparations are required).Concentrate on technical topics.

1. Thyristor is a ……………device ans: pnpn2. LVDT works on the principle of……………3. A diameter of the wheel is 20 cm if it revolved around 60 revolutions then how many revolutions will be made by a wheel of 50 cm diameter.4. Whether compressor or the turbine should be started first in the case of a gas turbine5. Discharge Vs time graph is called as……………….6. x/y=2; x=4 then (x-y)/y=?7. p/q=2;what is the value of 2p/q+q ans : can’t be determined8. What is the cutin voltage of germanium and silicon diodes??

Theory questions from the following areas were asked.

9. Torque Vs speed characteristics of shunt motor10. Dynamic breaking11. Basics in Electron devices and Induction and Synchronous Machines

Problems on 

12. Maximum Power Transfer theorem13. Star to Delta Conversion.14. Network Theory

SCT Sample test Paper

1. Rs.1260 shared amonng A,B,C in the ratio 2:3:4.What is the C's Share?

Ans: (4/9)*1260 =560

2. There are 2 cups one half filled with tea and other half filled with coffee.If 1 tea

spoon of tea is takebn and mixed with coffee cup & 1 tea spoon from thecoffee mixture is

Page 13: 7116273 Aptitude IGATE Infosys Tcs Ect

taken is mixed with tea cup.Which has higher concentration?

Ans: Tea

3. Jack is taller than Peter.Bill is Shorter than Jack.

a) Bill is Taller than Peter

b) Bill is Shorter than Peter

c) Bill is as Tall as Peter

d) imposssible to say

Ans: d

4. BGLQ:YDIN:VBGL:?

Ans: EJOT

5. A Room has round table at one corner.The one edge of the table is 5 inch from the

wall and the other edge is 10 inch from the other end of the wall.What is the diameter of

the table.

6. A passage is given in a jumbled manner.Rearrange and give a suitable heading.

Think soft Sample test paper

 Patern:            60 questions 45 minutes

Reading Comprehension

Aptitude(Maths)

Synonyms

Antonyms

Find Grammatical Mistakes

Data Sufficiency

Synonyms 

1. Mandatory - compulsory

2. affirmed - confirmed

3. illicit - illegal

4. capricious - whimsical

5. alieviate - relieve,remove

Data Sufficiency

1.  

1. Two triangles are congruent

2. two triangles are right triangles

Page 14: 7116273 Aptitude IGATE Infosys Tcs Ect

3. two triangles have same perimeter

Answers

1. statement 1 is enough

2. statement 2 is enough

3. statement 1 and statement 2 together is required

4. Statement 1 is enough and statement 2 is enough separately

5. data is insufficient

2.       y > 0

      y square 2 - 4 > 0

      Answers:

    1.statement 1 is enough

    2.statement 2 is enough

    3.statement 1 and statement 2 together is required

    4.Statement 1 is enough and statement 2 is enough separately

    5.data is insufficient

Aptitude

1. 37.5 % of a number is 450 what is the 87.5 % of the number?

Ans:1050

2. 30% of 40% of 200

Ans:24

3. SI = 240

R = 6

N = 4

P = ?

    Ans:1000

4. diff bet. simple interest and compound interest for 2 years = p(N/100) square 2  A

problem based on this.

XansaSection 1 - Quantitative

1. Sum of three nos is 98. The ratio between 1 and 2 is 2:3. The ratio between 2 and 3 is

5:8 . Find the second no?

2.  A car travels uphill at 30 km/hr and downhill at 60 km/hr. It goes 100 km uphill and 50 km

downhill. Find the average speed of the car?

3. A batsman's avg in 12 innings is 24.00 . If his avg is to be double of the no of innings (15

innnings), what  should he score in the remaining three innings (avg)?                                

4. A man buys 1kg of sandalwood and 1kg of teakwood. He sells one for 10% profit and other

for 10% loss.What is total profit/loss percentage?

Page 15: 7116273 Aptitude IGATE Infosys Tcs Ect

5.  In a class of 250 students, on JAN 2 15% of the girls and 10% of the boys are absent. If on

100% attendance there are 10 boys. Find the percentage present?

6.  Mandrake has to choose from 4 from 10 people. There are 3 girls, 5 boys , 2 children. What

is total probability that he will choose 1G , 2B , 1C?

7. Some questions on Geometry?

8. Some questions on Measurements?

9. Some questions on permutation and combination.

       Section 2 - Ananlytical

10. Cities A - M are connected.Distance between any two cities is 1mile.They gave which city

connects which city.Q's based on these data  

11.  One pblm on DI.Gulshan Kumar and Rakesh Roshan take a film. If they take a art film it's 25

lacs or a multi starrer it's 75 lacs.They spend 28% on clothing ,11% on lighting.Actor fee 22% for

Multi starrer or 10 lacs for art film.Based on these data some questions were asked.

       Section 3 - Verbal

       General GRE type q's.:Analogies,Reading Comprehension.                        

      Section 4 - Visual reasoning.

 

      Some figures were given and told to fill up the blanks. 

     Some figures were given and told to fill up the blanks. for the post of system trainee. They need people

to work in MAINFRAMES.The paper was ok, except a few questions others were basic.They paper consisted

of 4 sections. Quants,Anals,Verbal and Visual Reasoning.They wnated us to attend all the sections, because

they clearly stated that any ommission, they will not value the papers and chances of getting shortlisted

was little. After the test they asked us to write a essay on any of these  three topics.

 My Family

My Career

What two things i would change if I was the prime minister of India.

   INTERVIEW:

   The tech interview was a breeze

   Tell me abt urself

Some questions on my project.

Q's from subjects studied in past semesters.He actually told me to write 4 important subjects

in each semester and asked q's from them.

BELThe interview for Probationary Engineer at BEL, Bangalore . 21st August, 2004

 Its for CSE stream. Interview lasts for 40 minutes. As I have to face my ever first Job interview, am with little bit anxiety. Here are the details of the interview I would like to share with these magnificent group friends.As I entered in to the room by asking excuse me Sir, They invited me very politely and offered a chair to seat. There were four members in the panel all were in age group of 50, of which two members are from Central Research Laboratory of BEL grilled out me with tech questions. One is HR department person and remained was acts as head of the panel.

First the head of the panel introduced all the members and explained the procedure of the interview. First he starts by describing about the company for almost five minutes,  which makes me to relax and comfortable with the panel. After that he asked to tell about myself briefly for a minute. 

While I am in a way of describing myself, he also asked to point out the achievements, strong points, hobbies and regarding sports. Also asked for favorite subjects, and I told that C, Database and Networks are my favorites.Now it’s the time for technical grilling..The first person from CRL  starts by asking

What are the components of LAN? 

How do u connects the systems?

What is UTP? (Unshielded Twisted Pair)

How much distance it supports?

Fiber optic supports how much distance?(I don’t know exactly?)

Page 16: 7116273 Aptitude IGATE Infosys Tcs Ect

What is the throughput of the Ethernet LAN?

Describe about OSI Layers?

In which layer encryption can be done? And other than presentation layer?

What is the function of Bridge? Router?

Which layer is responsible for end 2 end connection?

What is RAID? (Redundant Array of Inexpensive Disks)

For what it is used for?

How can achieve fault tolerance through RAID?

Where the Static variable stored in C?

And local variables?( I was made a mistake here by telling it is also in heap)

And he shows already written function prototypes asked to describe it.. as             int *(*p)(int a, float b) // pointer 2 the function p returns integer pointer               int  (*p)(int *a, float b) //pointer 2 the function  p returns integer               int  *p(int a, float *b)   // function p returns integer pointer               int  p(int a, float *b)    // function p returns integernow he looks as impressed and asked

how to return 1000 variables from a function in C?

Now seen changes to second person of CRL

He asked to explain about my project? (It lasts for 4 minutes)

What are the functions of the OS?

Which OS you are familiar with?

What is the process?

What r its attributes?

How the process interacts with each others?

What r the mechanisms for inter process communication?

What is semaphore?

What is critical region?

How can semaphore avoid conflicts?

Again first person starts asking 

What is a virtual function?

What is late binding?

What is scope resolution operator?

Differentiate between testing and debugging? (here I take time 2 distinguish )

Distinguish black box testing and white box testing?

What is unit testing?

How can u control the lost acknowledgement issue?

Is there is any difference between switch and hub? Which is good? ( here again I wasn’t answered properly)

And some more questions I wasn’t reminded correctly..

After that head of the panel gestures to HR to ask questions

What do u prefer to be a team leader or team member?

What r the qualities u have for being a leader?

Page 17: 7116273 Aptitude IGATE Infosys Tcs Ect

How can u solve the conflicts in a team?

How can u react if some one else competing with u in ur

team for promotion?

Again head asks me

What is your reaction if some one copying in the exam?Can u tell me a instance in your college that insists your leadership attitude?

Have u ever short tempered?

Have u attended any interviews?

Is u don’t have campus?

And he gestures to HR to explain pay and bond details

He told that first 6 months will be the  training period after that for three years u will be appointed as probationary engineer, and also told that based on our performance promotion will be carried out early or late.

For this probationary engineer u can draw 2.65 laks per anum with all allowances, and u have to execute a bond of Rs50000/- for three years effects from date of appointment after training.

And now head asks for any questions from me. I was asked the same qns 

What is the career path?

When can I know the results?

Why u were not strong in media advertisements? Blah blah bla

Kanbay

The selection took place in three rounds each of which was an elimination round.

1)Written Test

2)Group Discussion

3)Personal Interviews

Each section contained 30 questions.The cut-off was 12 marks in each section which was clearly announced

before the exam started.1 Mark was awarded for each correct answer.1/4 Mark was deducted for each

wrong answe                                                                         

The questions given below were all that could be reproduced from all the 3 sets together.

1. Average of 5 innings is 20. If four innings are 32, 10, 40 and 12 find the 5th inning score.

2. Area of path around a square is 256 sq.m. Find the width of path if the side of square is

30m.

3. A company increases the bonus 3 times of its 50 employees. What is the % increase in

bonus?

4. A man swims in a river 5 min upstream and 5 min downstream. Starting at A and ending at

B. What is the speed of the man?

5. NESTIN is a jumbled word. What are the first and last letters of the correct word.

6. There is enough material to fence 30m linear. Also there is old fence on oneside of the plot.

Find the length and breadth of the plot.

7. In a game of Snooker A gives B 18 points for 90, A gives C 10 points for 60. How many can

C give B in a game of 70?

8. C is the daughter of B. h is the brother of C. F is the father of C. G is the son of F. D is the

brother of F. What is the relationship between C and D?

Page 18: 7116273 Aptitude IGATE Infosys Tcs Ect

9. A moves 25km North and then finds that she is movin the wrong way. She then takes a

right turn and moves 2km. Then again she takes a right turn and moves 25km. How much distance

does she have to travel to now reach the starting point?

10. Some cats are good. Some cats are black.                                                           

11. Gold is 19 times heavier than water. Copper is 9 times heavier than water. In what ratio

should they be mixed so that the alloy is 15 times heavier than water?

12. A sphere of radius r is placed inside a cylinder of height 2r which just fits. What is the

Volume of the empty space?

13. A is 3 times as old as B. B is 4 years older than C. If C is Z years old express A in terms of Z.

14. A is punctual. All punctual are on time.

            Conclusion to be chosen from the given multiple choises.

15. a_ _ _baccba_ _ _aa

16. Srikanth has borrowed Rs 3000 at 10% C.I.. He has to repay that after 3 years in 3 equal

installments. How much does he have to pay as an installment?

17. Given a date and day, the question was to calculate the day of the date June 12, 1979.

18. The weight of a Bucket full of water is 17 kg, the weight of the same bucket with half full

water is 13.5 kg. What is the weight of the Bucket?

19. In a party attended by boys and girls where the girls outnumbered the boys Rs 280 was

distributed. Each boy was given Rs 10 and each girl Rs 20. How many boys are there?

20. 10% of an army died. 10% of the were ill. 12% of the rest met with an accident. What is the

total number of the original army if 7 lakhs were finally left?                                                  

21. A started a business with Rs 1700 amount. B joined A after 3 months , C joined after 6

months. If the profit is Rs 1700 and they shared it in the Ratio of 2:3:5 how much did B and C

invest?

22. A beats B by 24m. A beats C by 20m. C beats B by 1 sec. In how many seconds does A

complete the race of distance 120m.

23. VIJAY is coded as YLMDB. STOP is coded as VWRS. 

A few questions were given based on this code.

24. 9 3 6

2 ? 1

4 7 8

25. 20 15 10

15 10 5

10 ? 10

26. Which is larger Rs 35 : Rs 1.40 or Rs 48 : Rs 1.44

27. A started a business with Rs 1700 amount. B joined A after 3 months , C joined after 6

months. If the profit is Rs 1700 and they shared it in the Ratio of 2:3:5 how much did B and C

invest?

28. In a group of people 600 are Non-veg and 400 are Veg. Of these 150 are both Non-Veg and

Veg. In the Non-Veg 300 eat Mutton and 400 eat Chicken.(i) Find the number of people who eat

only Non-Veg Mutton.(ii) Who eat only Veg.

29. In a wall clock the smaller needle is between 8 and 9 and the larger makes a right angle

with it. Find the exact time.

30. If the temperature increases uniformly from 9 am to 2 pm and it increases from 21 C to 36

C. What is the temperature at noon?

Page 19: 7116273 Aptitude IGATE Infosys Tcs Ect

31. Place a word in the brackets such that it makes a meaningful suffix to the first part and a

meaningful prefix to the second part. 

TEM (_ _ _ _) ERS                                                                                      

32. A monkey climbs 3m of a greased pole in 1 minute and then slips 1m in the next. When will

he reach the top of the pole of 12 m?

33. The Alphabet that is 3 from the right to the letter between K and S.

34. Length and Breadth of a Rectangle are 100 m and 6 m. Find the area of a road around the

rectangle with a width of 5m.

35. Find the figure with the highest Area

i)Circle with r=2.

ii)Equilateral Triangle with Side = 4.

iii)Square of diagonal =2.

iv)Triangle with a=8, b=5 and c=4.

36. A boat moves 10 km upstream and returns to the same point in 45minutes. The speed of

the stream is 3km/hr. The speed of the boat is ?

37. Given an Equilateral triangle of side 10m. A cow is tied to one end. The length of the rope is

7 m. The area covered by the cow is ?

38. A is 7th from the left, B is 9th from the right, if they exchange A is 11th from the left. The

total number of persons is?

39. A person moves 5 km East, turns right moves 4 km and then turns right again and moves 5

km. What is the position of the person from the starting point?

40. If PLEASE is coded as RMIGKI, then SLEEP is coded as ______

41. If TRUCK = 25 and DERIVE = 36 , What is CAPTAIN ?

42. A rectangular field of length 30 m and breadth 18 m. A carpet of 20 cm width is to be used

to cover the field. The carpet costs 0.50ps per metre. What is the total cost of carpeting?

43. A person travels 20km towards North, he takes left and travels for 15km, then he takes left

and travels for 20km. In what direction is the travelling now?                                                     

44. A train goes from A to B at 50 km/hr and comes back at 40 km/hr and hence takes an hour

more time to return. What is the distance between A and B?

Q45)D/23, 17/F, I/12, 8/M, R/5

Nucles

Aptitude Section Q Page 1 of 8

Aptitude Section

Q. 5 men or 8 women do equal amount of work in a day. a job requires 3 men and 5 women to finish the job

in 10 days how many woman are required to finish the job in 14 days.

a) 10

b) 7

c) 6

d) 12

Ans 7

Q. A simple interest amount of rs 5000 for six month is rs 200. what is the anual rate of interest?

a) 10%

b) 6%

c) 8%

Page 20: 7116273 Aptitude IGATE Infosys Tcs Ect

d) 9%

Ans 8%

Q. In objective test a correct ans score 4 marks and on a wrong ans 2 marks are ---. a student score 480

marks from 150 question. how many ans were correct?

a) 120

b) 130

c) 110

d) 150

Ans130.

Q. An artical sold at amount of 50% the net sale price is rs 425 .what is the list price of the artical?

a) 500

b) 488

c) 480

d) 510

Ans 500

Technical Section

Q. You are creating a Index on EMPNO column in the EMPLOYEE table. Which statement will you use?

a) CREATE INdEX emp_empno_idx ON employee, empno;

b) CREATE INdEX emp_empno_idx FOR employee, empno;

c) CREATE INdEX emp_empno_idx ON employee(empno);

d) CREATE emp_empno_idx INdEX ON employee(empno);

Ans. c

Q. Which program construct must return a value?

a) Package

b) Function

c) Anonymous block

d) Stored Procedure

e) Application Procedure

Ans. b

Q. Which Statement would you use to remove the EMPLOYEE_Id_PK PRIMARY KEY constraint and all

depending constraints fromthe EMPLOYEE table?

a) ALTER TABLE employee dROP PRIMARY KEY CASCAdE;

b) ALTER TABLE employee dELETE PRIMARY KEY CASCAdE;

c) MOdIFY TABLE employee dROP CONSTRAINT employee_id_pk CASCAdE;

d) ALTER TABLE employee dROP PRIMARY KEY employee_id_pk CASCAdE;

e) MOdIFY TABLE employee dELETE PRIMARY KEY employee_id_pk CASCAdE;

Ans. a

Q. Which three commands cause a transaction to end? (Chosse three)

a) ALTER

b) GRANT

c) DELETE

d) INSERT

e) UPdATE

f) ROLLBACK

Ans. a ,b ,f

Page 21: 7116273 Aptitude IGATE Infosys Tcs Ect

Q. Under which circumstance should you create an index on a table?

a) The table is small.

b) The table is updated frequently.

c) A columns values are static and contain a narrow range of values

d) Two columns are consistently used in the WHERE clause join condition of SELECT statements.

Ans.d

Q. What is the common standard naming convention of checkbox control?

a) CHB

b) CHKc) CHX

d) CBX

Q. Which of the function returns a reference to an object provided by an ActiveX component.

a) createobject

b) getobjectname

c) createobjectx

d) getobject

Q. We have something like Global functions in JAVA, they are called as .....

a) class

b) package

c) file

d) include

Q. Which all OS supports Networking?

a) Windows 95

b) Linux

c) Windows 3.0

d) Unix

Q. Which of the following is not an RdBMS?

a) Ingres

b) Oracle

c) Unify

d) Clipper

Q. Shell function in VB is used for calling

a) Another Function

b) Another Procedure

c) Another Application

d) None

Q. The RdBMS which satisfies the most number of its Principle among the followings

a) MS SqlServer

b) Oracle 7.3

c) Informix

d) Sybase

Q. Normalization is considered to be complete when it is in

a) Second Form

b) Third Form

c) First Form

d) None

Page 22: 7116273 Aptitude IGATE Infosys Tcs Ect

Q. Two databases can be connected with

a) Where Clause

b) creating link

c) using dbo.<database>

d) Both B & C

Q. C++ is similar to that of C in following ways

a) C++ has classes

b) Supports Inheritance

c) File Handling

d) None

Q. Which of the following is not system file.

a) .ini

b) .sys

c) .com

d) None

Q. Following command is used to register any dll or ocx in registry of the system

a) regserver32

b) registersvr

c) regsrv32

d) regsvr32

Q. Which keyword is used to unregister any dll or ocx in registry of the system

a) -u

b) -r

c) -d

d) -x

Aptitude Section Q Page 5 of 8

Q. Which is not the most important & widely used form of Normalization ?

a) Boyce-Codd Normal Form

b) Second Form

c) Third Form

d) Royce-Codd Normal Form

Q. How can the word YES be stored in any array.

a)array[1] = 'Y'

   array[2] = 'E'

   array[3] = 'S'

   array[4] = '\0'

b) array[0] = "Y"

    array[1] = "E"

    array[2] = "S"

    array[3] = "\0"

c)

   array[1] = "Y"

   array[2] = "E"

   array[3] = "S"

Page 23: 7116273 Aptitude IGATE Infosys Tcs Ect

d)array[0] = 'Y'

   array[1] = 'E'

   array[2] = 'S'

   array[3] = '\0'

Q. Which of the following keyword is used to exit unconditionally from the batch?

a) go

b) return

c) Begin & End

d) Commit Tran

Q. != is a ---------- operator.

a) relational

b) logical

c) String

d) arithmetic

Q. What was the first name given to Java Programming Language. Aptitude Section Q Page 6 of 8

a) Oak - Java

b) Small Talk

c) Oak

d) None

Ans.a

Q. The syntax of Java is similar to that of

a) C

b) Small Talk

c) FORTRAN

d) C++

Q. Which of the following statement is true Table in a database can have

a) One Non-Clustered Index and Many Clustered Indexes.

b) One Clustered Index and Many Non-Clustered Indexes.

c) One Index each of Clustered and Non-Clustered Index.

d) None

Q. Check the error in the following statement Country[7] = 'CANADA'

a) A string terminator is not added to the string, when declared.

b) Country array should be of six

c) Canada should be specified in double quotes.

d) Country array should have the keyword char to ensure array type.

Q. An application updates table "A",which causes trigger T1 to fire. T1 updates table "B", which in turns fires

trigger T2. T2 updates table "A", which causes trigger T1 to fire again. This is an example of

a) Indirect Recursive Trigger

b) direct Recursive Trigger

c) Multiple Trigger

d) Non Recursive Trigger

Q. Linda wants to obtain the nearest integer of a numeric expression for some calculation purpose.

Which mathematical function will she use:

a) Round

b) ABS

Page 24: 7116273 Aptitude IGATE Infosys Tcs Ect

c) About

d) None

Q. Alphanumeric constants are

a) used for arithmetic calculations

b) Used with double quotas

c) Of integer type or float type

d) Not used for arithmetic calculations

Q. Pseudocode is a

a) set of Instructions to perform a particular task

b) is a formalized graphic representation of program logic.

c) is a algorithm expressed in a simple language

d) Both A & C

Q. A company has closed down its advertisement dept and is now getting all advertisement done by an Ad-

Agency. All 20 people working in the dept has quit the job. The dept to which an employee belonged was

stored in the "cdept" attribute of "emp" table. Which of the following statement would be used to do the

changes in the "emp" table

a) Alter Table

b) Drop Table

c) Delete Table

d) Truncate Table

Q. John wants to retrieve all records from students table who live in any city beginning with WAS . Which of

the following statement is to be executed by him

a) Select * from students where city = 'WAS'

b) Select * from students where city = 'WAS%'

c) Select * from students where city in 'WAS'

d) Select * from students where city like 'WAS%'

Q. Why is a Modulo operator used?

a) It is used to determined the remainder, when an integer is divided by another.

b) It is used to calculate the percentage

c) It is used to determine the factorial of a number.

d) It is used as a relational operator.

Q. Consider the following program:

Aptitude Section Q Page 8 of 8

character cName[5] = 'great'

Numeric nNum1,nNum2 =0

For (nNum1 = 0;nNum1=>5;nNum1++)

{

if(cName[nNum1] == 'a'| cName[nNum1] != 'e'| cName[nNum1] = = 'i'| cName[nNum1] != 'o'|

cName[nNum1] == 'u'|)

{

nNum2 ++

display nNum2

What does nNum2 display.

a) 2

b) 1

c) 5

d) 3

Novartis

Page 25: 7116273 Aptitude IGATE Infosys Tcs Ect

CATEGORY : JAVA - J2EE (3+ years experienced category)

About Company: NOVARTIS is a Swiss based MNC and world number one in Pharmaceutical. It has a very good brand name. It's having around 80,000 employees. It's entering IT insustry now to handle the large number of their in-house projects. As of now, the only development center in India is in Mumbai. The interviews are held for more than 3 years experience in JAVA, J2EE. 

There are 3 rounds :

1. Technical Test (25 questions in 45 minutes time)2. Group Discussion (30 minutes) 3. Technical & HR interview (30 minutes to 90 minutes)I wrote here all the questions I remember. I think I almost covered most of them.______________________Technical Test______________________All are multiple choice questions.1) Question on Static Methods, whether they can be overloaded or not2) A java program on nested (inner) loops and it is asked what is the output of the program.3) Once a Servlet is initialized, how do you get the initialization parameters ?(a) Initialization parameters will not be stored(b) They will be stored in instance variables(c) using config.getInitParameters()ANS: I think answer is (c)

4) A question on functionality of <forward> tag in JSP5) If the cookies are disabled, how can you maintain the Session.ANS: URL rewriting6) If there are strict timelines and if you want to get high performance for JSP to DB access, what method you suggest ?(a) Moving application server in to same manchine as Database(b) By storing results in Cache(c) By implementing Connection PoolingANS: I think answer is (c)

7) A question on MVC architecture and the functionality of Controller and View. 8) Question on Design Pattern. (I don't remember it)9) Which Design Pattern hides the complexities of all sub-systems ?(I don't remember the options and also don't know answer.)10) In CMP bean, which method executes only once in life time(a) setEntityContext()(b) create()(c) remove()(d) find() ANS: I think answer is (b)11) Which bean can be called as Multi-Threaded bean ?(a) Entity beans(b) Stateless Session beans(c) Stateful Session beans(d) Pooled Stateless Session beansANS: I think answer is (d)

12) A question on Threads in Java, whether we need to mention the word "Daemon" explicitly to make a thread as Daemon.13) A question on Transactions of EJB. I think the question is something similar to - "Which is faster ?"(a) TRANSACTION_UNREPEATABLE_READ(b) TRANSACTION_REPEATABLE_READ(c) TRANSACTION_COMMIT_READ(d) TRANSACTION_UNCOMMIT_READ(I don't know answer and also I am not sure of options. but the options are something similar to this.)14) Question on EJB Home Object, Remote Object and what functionalities will be performed by each.15) What is the difference between Server and Container(a) A Container can have multiple Servers(b) A Server can have multiple Containers(c) A Server can have only one ContainerANS: I think answer is (b)16) ejbStore() method is equivalent to(a) SELECT(b) INSERT(c) UPDATE(d) DELETEANS: I think answer is (c)17) A question on where the garbage collection is done. I think the answer is : "finalize()" method18) A question properties of Primary key in Entity Beans (I don't remember the options exactly.)(a) Primary key consists of only basic data types in java(b) Primary key can contain composite data typesRemarks on Technical Test : It's a bit difficult and lot of questions are on EJBs, JSPs and Design Patterns. Group Discussion

Page 26: 7116273 Aptitude IGATE Infosys Tcs Ect

Topics:1. Development of India2. Qualities to become a successful manager__________________________Technical & HR Interview__________________________

1) Tell about yourself ?

2) Explain your projects and what design patterns they follow !

3) Questions on Project management, Team management, Defect prevention, Quality procedures (These, questions are in detail and on each aspect. This went on for around 1 hour.)

4) Tell something about your current company

5) Reasons for leaving current company.

6) Current salary & Expected salary !

6) Any Questions ?

Remarks on Technical & HR Interview : Be prepared well for projects you did and the Design Patterns in J2EE. Also the Quality procedures.

QuarkQuark TEST PAPER

Aptitude and Computer Awareness No Negative Marking.45 minutes to complete the test. Do not write anything on question paper. 

QUANTITATIVE APTITUDE Directions for question nos. 1-2: Eighty Five children went to amusement park where they could ride on merry –go-round roller coaster and Ferris wheel .It was known that 20 of them have took all three rides and 55 of them have taken at least two of the three rides. Each ride cost Rs.1 and the total receipt of the amusement park was Rs.145.

1) How many children did not try any of the rides. ?A) 5 B) 10 C) 15 D) 20  Ans. 15.20 kids * 3 rides = Rs. 60(55-20=)35 kids * 2 rides = Rs. 7060 + 70 = Rs. 130So, Rs. (145 – 130 = ) 15 are left for the other (85 – 55 = ) 30 kids . so only 15 of them can take a ride and rest 15 will be left out.145 rides were taken. 20 of them took all three, i.e. Rs. 60 were spent, so 145-60= Rs. 85 are left for the others. Total kids were 85, so rest were 65. out of these 65, 

2) How many children took exactly one ride?A) 5 B) 10 C) 15 D) 20 Ans. 15

3) Four cities are connected by a road network as shown in the figure. In how many ways can you start from any city and come back to it without travelling on the same road more than once ?A) 8B) 12C) 16D) 20Ans. 12. Consider the top city, the following are the 3 routes possible, starting from the leftmost edge. Since there are 3 edges emanating from each city and the figure is perfectly symmetrical, these 3 routes are possible from each edge, hence for any given city, the total number of routes = 4 * 3 = 12. 

Directions for question nos 4-5:A, B, and C are three numbers, Let@(A, B)= Average of A and B*(A, B)=Product of A and B/(A, B)=A divided by B

4) If A=2 and B=4 the value of @( / (*(A,B),B),A) would be A) 2 B) 4 C) 6 D) 16Ans. 2 

5) Sum of A and B is given by

Page 27: 7116273 Aptitude IGATE Infosys Tcs Ect

A) *(@(A, B), 2) B) /(@(A,B),2) C) @(*(A,B),2) D) @(/(A,B),2Ans. A.

6) Let x<0, 0<y<1, Z>1 which of the following is false:A) (x2-z2)has to be positive.B) yz can be less than one.C) xy can never be zeroD) (y2-z2) is always negativeAns. A.

7) If A’s income is 25% less than B’s ,by what % is B’s income greater than that of A ?A) 35% B) 25% C) 30% D) None of theseAns. D.Directions for question nos 8-12: A professor keeps data on students tabulated by the sex and the performance of the student. Data is kept in a computer disk, but unfortunately some of it is lost because of a virus. Only the following could be recovered:Performance TotalAverage Good Excellent Male 10 Female 32Total 30 

Panic buttons were pressed but to no avail. An expert committee was formed,which decided that the following facts were self evident:a) Half the students were either good or excellent.b) 40% of the students were femalec) One third of male students were average

8) How many students were both female and excellent?A) 0 B) 8 C) 16 D) 32Ans. A

Performance TotalAverage Good Excellent Male 10 48Female 32Total 30 80

Panic buttons were pressed but to no avail. An expert committee was formed,which decided that the following facts were self evident:d) Half the students were either good or excellent. (implies the total of good and excellent is 40, i.e., hall of 80)e) 40% of the students were female (this implies that 32 is 40% of the total students, hence total students are 80, this implies that males are 48, calculate the rest yourself)f) One third of male students were average

9) What proportion of good students are male?A) 0 B) 0.73 C) 0.4 D) 1.0Ans. B.

10) What proportion of female students are good?A) 0 B) 0.25 C) 0.50 D) 1.0Ans. B

11) How many students are both male and good?A) 10 B) 16 C) 22 D) 48Ans. C

12) Among average students, what is the ratio of male to female?A) 1:2 

Page 28: 7116273 Aptitude IGATE Infosys Tcs Ect

B) 2:1 C) 3:2 D) 2:3Ans. D

ANALYTICAL

Directions for Questions Nos: 13 to 17Five executives of a multinational company met in Bombay:Mr. Ram can speak Tamil and HindiMr.Sham speaks Tamil and EnglishMr. Raju converses in English and HindiMr.Balu speaks Telugu and Tamil quite well.Mr.Lalu can speak Hindi and Telugu

13) Which of the following can act as a interpreter when Mr.Raju and Mr.Balu wish to confer?A) Mr.Ram OnlyB) Mr.Sham Only.C) Either Mr.Ram or Mr.ShamD) Any of the other three executives.Ans. D 14) Besides Mr. Lalu, which of the following can converse with Mr.Balu without an Interpreter?A) Mr.Ram OnlyB) Mr.Sham onlyC) Mr.Ram and Mr.ShamD) Mr.Raju OnlyAns. C

15) Which of the following cannot converse without interpreter?A) Mr. Sham and Mr.Lalu.B) Mr. Ram and Mr. ShamC) Mr. Ram and Mr. RajuD) Mr. Sham and Mr. BaluAns. A 16) If a sixth executive is brought in, for him to be understood by the maximum number of original five, he should be fluent in?A) English and TeluguB) Hindi and TamilC) Telugu and HindiD) Hindi and EnglishAns. B 17) Of the languages spoken the most common languages areA) English and TamilB) English and HindiC) English and TeluguD) Hindi and TamilAns. D Directions for Questions Nos:18 to 21Four people of different nationalities live on the same side of a street in four houses each of different color. Each person has a different favorite drink. The following additional information also known:The Englishman lives in the red house.The Italian drinks tea.The Norwegian lives in the first house on the left.In the second house from the right they drink milkThe Norwegian lives adjacent to the blue houseThe Spaniard drinks fruit juiceTea is drunk in the blue house.The White House is to the right of the red house 18) Milk is drunk byA) NorwegianB) EnglishmanC) ItalianD) None of theseAns. B

19) The Norwegian drinksA) MilkB) CocoaC) TeaD) Fruit JuiceAns. B

20) The color of Norwegian’s house isA) YellowB) WhiteC) BlueD) RedAns. A

Page 29: 7116273 Aptitude IGATE Infosys Tcs Ect

21) Which of the following is not true:A) Milk is drunk in the red houseB) Italian lives in the blue houseC) The Spaniard lives in a corner houseD) The Italian lives next to Spaniard.Ans. D Directions for Questions Nos: 22 to 23Kya-Kya is an island in the south pacific .The inhabitants of Kya-Kya always answer any question with two sentences, one of which is always true and other is always false.22) You are walking on a road and come to a fork. You ask the inhabitants Ram, Laxman, Lila,”Which road will take me to the village?”Ram says,”I never speak to strangers. I am new to this place” Laxman says,”I am married to Lila. Take the left road”.Lila says,”I am married to ram. He is not new to this place”Which of the following is true?A) Left road takes you to the villageB) Right road takes you to the villageC) Lila is married to laxmanD) None of aboveAns. A.

Ram said he never talked to strangers, but he spoke to a stranger, this means that this statement is false, hence his other statement must be true, hence the second statement of Lila is false, hence her first statement is true that is she is married to ram, hence the first statement of Laxman is false, hence his second statement is true, that is take the left road.

23) You find that your boat is stolen.You question three inhabitants of the island and they reply as follows:John says,”I didn’t do it. Mathew didn’t do it”Matthew says,”I didn’t do it. Krishna didn’t do it.”Krishna says,”I didn’t do it .I don’t know who did it.”Who stole your boat?A) JohnB) MatthewC) KrishnaD) None of them.Ans. BMatthew said he didn’t know who did it, but he also said he didn’t do it, which means he knows who did it, which means his second statement is false, which means his first statement is true…Directions for question nos 24-25: There are five trains A, B, C, D and E that run between the following stations:Bombay and PuneCalcutta and BombayPune and GoaGoa and BombayPune and CalcuttaTrains A and D do not go to Bombay, B&C do not go to Calcutta and C & D do not touch Goa.

24) Train E goes from………… to………… and vice-versa

A.Pune and Calcutta B.Bombay to Goa C.Bombay to Calcutta D.Goa to Pune.Ans. C

25) Which train runs between Calcutta and Pune?A) A B) B C) C D) DAns. D

COMPUTER AWARENESS

26) The device that can transform digital data into analog data is called a a)transformer b)network c)carrier d)modem Ans. d

27) Following is true about the IP of a machine a) It is 48-bit and will always be unique around the world. b) It is 48-bit and is not necessarily be unique around the world. c) It is 32-bit and will always be unique around the world. d) It is 32-bit and is not necessarily be unique around the world. Ans. c

Page 30: 7116273 Aptitude IGATE Infosys Tcs Ect

28) A machine having 64MB memory runs a executable which is 300MB on disk. This is achieved by: a) Use of FAR pointers b) Page swapping. c) Save some variables on another machine on network. d.) Cannot be run on the machine. Ans. b

29) Which of the following is true about thread and process startup speed: a.) The startup of a thread is faster than a process. b.) The process startup is faster as it is directly controlled by the OS. c.) They will be equal. d.) Depends on OS that is used. Faster on Windows98 slower on NT. Ans. a

30) What causes "Thrashing" of a program : a.) The constant swapping of program due to page faults. b.) The inability of a program to get assess to a network resource. c.) A near overflow / underflow of a variable. d.) Assessing a memory area not allocated to the process. Ans. a

31) Turbo-C is a / an a.) IDE and C compiler/linker. b) C-compiler/linker c) C . d) code generator. Ans. a

32) The path of creation of an executable is : a.) coding, linking, compiling, parsing. b.) coding, parsing, compiling, linking. c.) coding, compiling, parsing, linking. d.) coding, compiling, linking, parsing. Ans. b

33) Memory leak in software is due to a.) Heavy recursion used in logic. b.) Using structures of large size c.) Improper use of the CPU registers. d.) Improper release of allocated memory . Ans. d

34) A "stable sort" is different from "sort" in the following way a) Stable sort handles multiple thread access. b.) Stable sort maintains the order of equal entities as it was in original sequence. c.) Stable sort will always sort using the fastest scheme available in the library. d.) Stable sort can handle exceptional conditions like interrupts in software. Ans. b

35) The classic way of checking whether a mathematical expression has matched paranthesis will employ the following data structure : a.) List. b.) Directed Graph c.) Threaded Binary tree. d.) Stack. Ans. d

36) The fastest sorting algorithm for a Random set of numbers is: a.) Quick sort b.) Shell sort c.) Bubble sort d.) Double Bubble sort. Ans. a or b, I don’t know

37) Which of these items is not a form of IPC: a.) Shared Memory b.) Pipes. c.) Message queues. d.) Semaphores Ans. b

38) The term "socket" in software refers to: a.) The software primitive which allows access to the hardware attached to the computer. b.) The API exposed by drivers to assess a hardware. c.) End point of connection used to transfer data programmatically. d.) Th primitive used by OS to get assess to the CPU for process scheduling. Ans. c

Page 31: 7116273 Aptitude IGATE Infosys Tcs Ect

39) Which of these statements is True: a.) XML is a sub-set of HTML. b.) XML is a less generic markup language standard derived from SGML. c.) XML is HTML for Xtended Interfaces like mobile-phones. d.) XML is a Xtension of HTML which defines new tags. Ans. d

40) DCOM and CORBA are: a.) Specifications which enable faster downloads on the net. b.) Specifications that allow objects to be accessed in a location independent manner. c.) Parallel implementations of XML by Microsoft and Sun respectively.. d.) Specifications to store objects on disk, for later retrieval. Ans. b

41) The Process that involves monitoring and improving the software product development is:(a) Quality Assurance(b) Quality Control(c) Quality Improvement(d) None of the aboveAns. 

42) An executable test that verifies a functionality of the software unit with given input and expected output is called:(a) Test Script(b) Test Plan(c) Test Case(d) All of the aboveAns. maybe c

43) A Bug in the software is, when there is(a) Application Crash(b) Feature Failure(c) Loss of Data(d) All of the aboveAns. maybe a, or probably d

44) Testing based on External Specifications without knowledge of how the system is constructed:(a) Black Box Testing(b) White Box Testing(c) Stress Testing(d) Performance Testing(e) None of the above Ans. a

45) Which of the following is false(a) QA is a process defined to attain Quality Standards(b) QC is testing of the product during its production workflow(c) A Test case could contain many Test Plans.(d) A Test Plan could contain many Test Script & Test CasesAns. c

46) A Printing Machine that transfers impressions from Flat Plate to Rubber Cylinder, thence to paper is called(a) Laser Printer(b) Press, Offset(c) Digital Printer(d) All of the aboveAns. maybe Offset

47) Printing process in which ink is applied to paper or board from raised portions of printing plates or type is called:(a) Printing, Letterpress(b) Printing, Flexographic (c) Printing, Silk screen (d) None of the aboveAns. maybe letterpress

48) Yellow (lemon), Magenta (cold red), Cyan (blue-green) are the three (a) Process Colors(b) Monitor Colors(c) Special Colors(d) Spot ColorsAns. 

49) Leading specifies:(a) The space between the lines in a paragraph.(b) The space between the base of a line to the base of the following line in a paragraph.(c) The space between the top of the X height and the bottom of the X height of the following line in a

Page 32: 7116273 Aptitude IGATE Infosys Tcs Ect

paragraph(d) The space between the beard of the top line and the beard of the bottom-line.Ans. 

50) Which is a typical page layout program out of the following software products:(a) Adobe Photoshop(b) Adobe PageMaker(c) Macromedia FreeHand(d) Macromedia Director

Ans. B

ANZPaper pattern  

 1.   analogies (from barron)- 8 ques.

 2.   technical section -os(most of the ques),ds(5-20),dbms(3-5),networks(1-2)

 3.   analytical/logical section -12 ques. (5ques. based on 1 ques.,5 ques. based on other,2

        ques. simple)

 4.   data sufficiency -20 ques. (practice from ims material)

 5.   c test - 5 ques. (easy)

     Quantities section  

1.   Given ip address detemine the class

2.   Terminal emulation (OSI LAYER) - application layer

3.   int a;

      scanf("%f",&a);  is there any error or warning ?

      ans. no compile time error but run time error

5.   Analytical ques. on rectangular table 8 persons sitting and some conditions (solved eg. in

barron)

6.   Analytical ques. based on table method given in barron (author names,publisher,book name)

7.   Marine animals,terrestial animals,amphibians - express it using venn diagram

8.   Some equations- solved by knowing 1st stmt,2nd stmt,both or none

9.   Difference between windows 3.1 and windows 95

10.  Some ques. on unix

11. Concurrency is NOT a feature of c++

12.  Ada is a stongly typed language 

       Interview tips  

    They have a lot of ques. on os

    Unix versions,virtual memory,processes and threads,

    Not asking dbms

    Some ques. on data structures

    Prepare ur project report properly

    For some full hr no technical

    They will respond strangely on your answers so don't deviate be confident on your answer

Page 33: 7116273 Aptitude IGATE Infosys Tcs Ect

ASDCSample Test Paper

   Section A: Aptitude

1. 0.03 of 0.05 is what 

a. 15%

b. .15%                                                                

c. 0.015%

d. 0.0015%

2. x<y,y<z and z>w which of the follwing is always true

 a. x<w

 b. y<w

 c. x>w

 d. y>w

3. 12 men can do a job in 4 hours .in wat time the same job can be done by 15 men

( I did not do)

a. 3 hrs

b. 3 hrs 24 min

c. 2 hrs

d. 3 hrs 30 min

4. 30 socks r there in a basket.60% are red and rest are blue. How many draws should

be taken from the basket to make sure that u have 2 blue and 1 red

a. 2

b. 3

c. 14                                                                             

d. 20

5. When operator * is applied to a number the result is 10 subtracted from the twice

of the original number ,so wat is *(*9)

6. A husband and wife has 6 sons and each has 5 children each, How many are there

in the family altogether

7. 3x-2y=8 so what is 4y-6x

 a. –16

 b. 16

 c. data not sufficient

8. From chennai to trichy it is 250 miles and from Chennai to pondicherry it is 120 miles.wat

percentage of mile from Chennai to trichy is Chennai to pondi

a. 34

Page 34: 7116273 Aptitude IGATE Infosys Tcs Ect

b. 20

c. 36

9. City B is 8 miles east of City A.City C is 6 miles north of City B. City D is 16 miles east of

city C. City E is north of City D by 12 miles .what is the shortest distance from City A to City E

10.  An employer pays X,Y,Z a weekly wage of total 610. X gets 120% of Y, and X gets 80% of

Z wat is the weekly wage of X   I think answer is 200.    

11. There are 50 employees of a company .21 were in training for both economics and science

training.11 were in 2 different training programs .find how many of them do not attend any

training programfew questions were given with some relationship like:

 1 head is to cap as finger is to

  Ring

 Nail

 Thumb

12.  In a certain code language RANGER was written as REGNAR .with the same code how is

TABLE written Elbat(reverse the word)

13. In a certain code if READ is written as SEADR then how is SING written with that code

SINGS

14. If kanchan is son of Sunil’s son’s son then how is Sunil related to Kanchan- grandson

15. Find the odd one out

a. Caution

b. Tresspasers will b prosecuted

c. Only one way

d. Keep left

16. One more odd man out was asked few questions on profit and loss were asked

 Technical

1. A sorting program is given .You need to print the output of every outer ‘for’ loop.

Main()

  {

  int n=8,I,j,k;

  int a[7]={44,55,42,12,6,94,67};                                      

  for(I=0;I<n-1;I++)

    {

     k=I;

    x=a[k];

    for(j=I+1;j<n;j++)

    {

     if(a[j]<k)

       {

         k=j;

         x=a[k];

         a[k]=a[I];

         a[I]=x;

       }

     }

}

2. The precedence of operators are given, you have to find the value  highest :+ followed by

-,*,/  parentheses r given higher precedence to operators unary operator ‘-‘ is always written

within parentheses  association is from right to left

3. Questions were given like L-Value R-Value

 a) z[I+2] 

 b)*x

Page 35: 7116273 Aptitude IGATE Infosys Tcs Ect

c)&x

d)&(&x)

e)

4. Assume the following:

a. A tape drive can store 4096 bytes/millimeter

b. Forgot

c. A IRG is necessary and the length is 2 millimeter                            

Answer the following question:

For storing 20MB how many millimeters of tape is required

What is the storage capacity if the tape is 10000meters long

For a infinite storage how many IRGs are required in a tape that stores 1 GB of data.

5. The following program is to calculate the number of lines, number of words, number of

characters from  file. You got to complete the C program (15 marks)

 # define IN 1

 # define OUT ------

        main()

       {

int nc=nw=nl= ------,state= ------.c;

while((----=getchar()!=EOF))

{

++ ----;                                                      

if (c= = ‘\n’)

++ ----;

if(c = = ‘ ‘ || c = = ‘\n’ || c= =’\t’)

state= ----;

elseif(state = ----)

{

state= -----;

++ -----;

}

}

}

Axes1. A 2MB PCM(pulse code modulation) has  

       a) 32 channels

       b) 30 voice channels & 1 signalling channel.                                          

       c) 31 voice channels & 1 signalling channel.

       d) 32 channels out of which 30 voice channels, 1 signalling channel, & 1 Synchronizatio channel.

       Ans: (c)

2. Time taken for 1 satellite hop in voice communication is

     a) 1/2 second

     b) 1 seconds

     c) 4 seconds

     d) 2 seconds

      Ans: (a)

3. Max number of satellite hops allowed in voice communication is :

       a) only one

       b) more han one

       c) two hops

Page 36: 7116273 Aptitude IGATE Infosys Tcs Ect

       d) four hops

       Ans: (c)

4. What is the max. decimal number that can be accomodated in a byte.           

       a) 128

       b) 256

       c) 255

       d) 512

       Ans: (c)

5. Conditional results after execution of an instruction in a micro processor is stored in

       a) register

       b) accumulator

       c) flag register

       d) flag register part of PSW(Program Status Word)

       Ans: (d)

6. Frequency at which VOICE is sampled is

       a) 4 Khz

       b) 8 Khz

      c) 16 Khz

      d) 64 Khz

      Ans: (a)

7. Line of Sight is

       a) Straight Line

       b) Parabolic

       c) Tx & Rx should be visible to each other                                            

       d) none

      Ans: (c)

8. Purpose of PC(Program Counter) in a MicroProcessor is

      a) To store address of TOS(Top Of Stack)

      b) To store address of next instruction to be executed.

      c) count the number of instructions.

      d) to store base address of the stack.

      Ans: (b) 

9. What action is taken when the processor under execution is interrupted by a non-maskable interrupt?

     a) Processor serves the interrupt request after completing the execution of the current instruction.

     b) Processor serves the interupt request after completing the current task.         

     c) Processor serves the interupt request immediately.

     d) Processor serving the interrupt request depends upon the priority of the current task under execution.

    Ans: (a)

10. The status of the Kernel is

       a) task

       b) process

       c) not defined.

       d) none of the above.

       Ans: (b)

11 What is the nominal voltage required in subscriber loop connected to local exchange?

          a) +48 volts

          b) -48 volts

Page 37: 7116273 Aptitude IGATE Infosys Tcs Ect

          c) 230 volts

          d) 110 volts

12. To send a data packet using datagram , connection will be established       

         a) before data transmission.

         b) connection is not established before data transmission.

         c) no connection is required.

         d) none of the above.

         Ans: (c)

13. Word allignment is

         a) alligning the address to the next word boundary of the machine.

         b) alligning to even boundary.

         c) alligning to word boundary.

         d) none of the above.

         Ans: (a)

14 When a 'C' function call is made, the order in which parameters passed to the function are pushed into

the stack is

        a) left to right

        b) right to left

        c) bigger variables are moved first than the smaller variales.                  

        d) smaller variables are moved first than the bigger ones.

        e) none of the above.

        Ans: (b)

15 What is the type of signalling used between two exchanges?

       a) inband

       b) common channel signalling

       c) any of the above

       d) none of the above.Ans: (a)

16. Buffering is

        a) the process of temporarily storing the data to allow for small variation in device speeds

        b) a method to reduce cross talks

        c) storage of data within transmitting medium until the receiver is ready to receive.

        d) a method to reduce routing overhead.

        Ans: (a)

17. A protocol is a set of rules governing a time sequence of events that must take place between

         a) peers

         b) non-peers

         c) allocated on stack

         d) assigned to registers.

18. Memory allocation of variables declared in a program is                           

         a) allocated in RAM.

         b) allocated in ROM.

         c) allocated on stack.

         d) assigned to registers.

         Ans: (c)

19. A software that allows a personal computer to pretend as a computer terminal is

          a) terminal adapter

          b) bulletin board

          c) modem

Page 38: 7116273 Aptitude IGATE Infosys Tcs Ect

          d) terminal emulation

          Ans: (d)

Blue StarBlue star paper has got 2 sections. Firstly u need to clear the aptitude test. It is very easy. Some of the

ques are as follows: 

    1.   Complete the series  1 6 3 7 5 8 7 ?    

              ans: 9

    2.   If circle is one octagon is 2 4 6 8 or 1                                         

    3.    pick the odd one out   bend   shave   chop   whittle   shear

            ans bend.

   4.    2 persons start from a pt. and go in opposite  directions. After going 3 km they turn left 

          and walk  4 km .how far r they now?   Ans: 10 km

  5.     There was a ques on work something like 2 typists type 2 papers in 4 hrs then in how much 

          time will 5 typists take or something like that

  6.     There were 2 ques on proverbs u were supposed to tell the meaning.they were easy.

  7.     A person runs 6 ft in 1/4 sec. how much it will run in 10 sec. ans 240

  8.     Find odd one cat ,dog,rabbit,hamster,elk.

  9.     In a party man & his wife ,and there r 2 son with their wives, and every son has 4 children

          in his family .how  manyperson r in party.

 10.    Unsreambled the word P U T E C A S A H (like that) paraschute

 11.    Add a word in starting of SCAPE and at the end of Grass, u will get two different word 

         ans:- land grassland landscape                                                                

 13.   All men of this party are republican . mohan is member of this party. mohan is republican is 

         true or false or camn't say?

 14.   There r 1200 elephant .some have blue and pink strips.some have pink & green strip. 400 ele.

         have only pink strip. how many have only blue strip   Ans:. 400

         b. can't determined. c.none

 15.   Day light Day break Day time After clearing aptitude there was technical paper.  It had 2

         sections.In section A there were 4 subsections out of  which 2 were to be attempted. the

          first subsection was on c and unix, second on c++,third on vb/asp/com and  fourth on rdbms. In 

          section B general computer awareness.  In section A I did c and c++. Some ques were

          as follows

                                         Technical  

   1.   void main()

         {

          extern int a;

          a=10;

           printf("%d",a);

          }

         1.  Give linker error- a not defined ans

          2.  Print 10

          3.  Give compiler error

   2.    int a[10];

         printf("%d,%d",a[0],a[12]);

         will compiler show any error?                                                                   

         ans no

  3.    socket() is a

           1.  system call ans

           2.   library function

           3.   both

           4.   none

Page 39: 7116273 Aptitude IGATE Infosys Tcs Ect

         In c++ there were conceptual ques on virtualfunctions , operator overloading etc. in section B 

          some ques were as follows 1.in microprocessor over clocking will result in

           1.  overheating

           2.  malfunctioning  ans(check)

           3.   life would be half

           4.

  4.    Cache is

           1.  ROM

           2.  RAM ans

           3.  PRAM

           4.  EPRAM

  5.      How can pass an array to a fns.                                                       

           1.   desgin(**a)

           2.   desgin (*a)

           3.  dasgien(&a[0]);

           4.

  6.     Unix has pipelining fromatted &

              a.  high level

              b.  low "

              c .  mid level

              d.  none "

  7.     Task switching is

             1.process switching

             2.

             3

             4.all of the above ans

  8.     Thread arev ans .light weight process+(c)....................... 

  9.     virual fns are defined as ................

 10.     which is not bus interface

            a.  pci

            b.  ide

            c.  age

            d.   none

 11.      switch statement use only                                                                           

            a.  char

            b.  int

            c.  both

            d.  all of the above

 12.    Multiple inheritence can not be used in a case.......................................?

 13.    What is difference b/w simple SQDL & Object oriented SQDL.

 14.    Virtual Destructor is used...........................

 15.    Question on two pass assembler.........................

 16.    Link list uses

            a.  countiguos memory allocation

            b.  non coun. " "

            c.  both

            d.  none

 17.    Non linear structure a. q & stack

            ans:- b.tree & graph

               c. tree & stack

               d. tree & linklist

Page 40: 7116273 Aptitude IGATE Infosys Tcs Ect

 18.    E ncryption is

             ans a. change readable code to cipher text

               b.none

                c.

                d.

 19.    Sizeof () operator is used for

          a.  data type

          b.  data type & veriable

          ans c. both

         d.   none

 20.    deadlock condition

          ans. no process knows exactly that both are waiting for each other.

 21.     static

          ans:- it's initial value is one --- not true

 22.     new can be overloaded ------ true

 23.     virtual function------- ans a and c means no code and forceful inheritance

 24.       for(;;)

           ans:- infinite loop

          Only three unix questions were there                                                           

          1. socket() is a answer: a) system call

      2. two types of pipeline in unix one is formatted  and other one   is highlevel middle level  

lowlevel none ( i don't answer for this question pls check  it out) RDBMS

1. CAN foreign key have null values?? 

answer: YES

2 IN RDbms SORTING IS PERFORMED IN   

ANSWER: TEMP

BPL

BPL Sample Question Paper

QUANTITATIVE SECTION

1. In a class composed of x girls and y boys what part of the class is composed of girls

A.y/(x + y)

B.x/xy

C.x/(x + y)

D.y/xy

Ans.C

2. What is the maximum number of half-pint bottles of cream that can be filled with a 4-gallon can of

cream(2 pt.=1 qt. and 4 qt.=1 gal)

A.16

B.24

C.30

D.64

Ans.D

Page 41: 7116273 Aptitude IGATE Infosys Tcs Ect

3. If the operation,^ is defined by the equation x ^ y = 2x + y,what is the value of a in 2 ^ a = a ^ 3

A.0

B.1

C.-1

D.4

Ans.B

4. A coffee shop blends 2 kinds of coffee,putting in 2 parts of a 33p. a gm. grade to 1 part of a 24p. a

gm.If the mixture is changed to 1 part of the 33p. a gm. to 2 parts of the less expensive grade,how

much will the shop save in blending 100 gms.

A.Rs.90

B.Rs.1.00

C.Rs.3.00

D.Rs.8.00

Ans.C

5. There are 200 questions on a 3 hr examination.Among these questions are 50 mathematics

problems.It is suggested that twice as much time be spent on each maths problem as for each other

question.How many minutes should be spent on mathematics problems

A.36

B.72

C.60

D.100

Ans.B

6. In a group of 15,7 have studied Latin, 8 have studied Greek, and 3 have not studied either.How many

of these studied both Latin and Greek

A.0

B.3

C.4

D.5

Ans.B

7. If 13 = 13w/(1-w) ,then (2w)2 =

A.1/4

B.1/2

C.1

D.2

Ans.C

8. If a and b are positive integers and (a-b)/3.5 = 4/7, then

(A) b < a

Page 42: 7116273 Aptitude IGATE Infosys Tcs Ect

(B) b > a

(C) b = a

(D) b >= a

Ans. A

9. In june a baseball team that played 60 games had won 30% of its game played. After a phenomenal

winning streak this team raised its average to 50% .How many games must the team have won in a

row to attain this average?

A. 12

B. 20

C. 24

D. 30

Ans. C

10. M men agree to purchase a gift for Rs. D. If three men drop out how much more will each have to

contribute towards the purchase of the gift/

A. D/(M-3)

B. MD/3

C. M/(D-3)

D. 3D/(M2-3M)

Ans. D

11. A company contracts to paint 3 houses. Mr.Brown can paint a house in 6 days while Mr.Black would

take 8 days and Mr.Blue 12 days. After 8 days Mr.Brown goes on vacation and Mr. Black begins to

work for a period of 6 days. How many days will it take Mr.Blue to complete the contract?

A. 7

B. 8

C. 11

D. 12

Ans.C

12.  2 hours after a freight train leaves Delhi a passenger train leaves the same station travelling in the

same direction at an average speed of 16 km/hr. After travelling 4 hrs the passenger train overtakes

the freight train. The average speed of the freight train was?

A. 30

B. 40

C.58

D. 60

Ans. B

13. If 9x-3y=12 and 3x-5y=7 then 6x-2y = ?

A.-5

B. 4

Page 43: 7116273 Aptitude IGATE Infosys Tcs Ect

C. 2

D. 8

Ans. D

ANALYTICAL ABILITY

1. The office staff of XYZ corporation presently consists of three bookeepers--A, B, C and 5 secretaries

D, E, F, G, H. The management is planning to open a new office in another city using 2 bookeepers

and 3 secretaries of the present staff . To do so they plan to seperate certain individuals who don't

function well together. The following guidelines were established to set up the new office.

I. Bookeepers A and C are constantly finding fault with one another and should not be sent together

to the new office as a team

II. C and E function well alone but not as a team , they should be seperated

III. D and G have not been on speaking terms and shouldn't go together

IV Since D and F have been competing for promotion they shouldn't be a team.

1.If A is to be moved as one of the bookeepers,which of the following cannot be a possible working

unit.

A.ABDEH

B.ABDGH

C.ABEFH

D.ABEGH

Ans.B

2.If C and F are moved to the new office,how many combinations are possible

A.1

B.2

C.3

D.4

Ans.A

3.If C is sent to the new office,which member of the staff cannot go with C

A.B

B.D

C.F

D.G

Ans.B

4.Under the guidelines developed,which of the following must go to the new office

A.B

B.D

C.E

D.G

Page 44: 7116273 Aptitude IGATE Infosys Tcs Ect

Ans.A

5.If D goes to the new office,which of the following is/are true

I.C cannot go

II.A cannot go

III.H must also go

A.I only

B.II only

C.I and II only

D.I and III only

Ans.D

2. 2.After months of talent searching for an administrative assistant to the president of the college the

field of applicants has been narrowed down to 5--A, B, C, D, E .It was announced that the finalist

would be chosen after a series of all-day group personal interviews were held.The examining

committee agreed upon the following procedure

I.The interviews will be held once a week

II.3 candidates will appear at any all-day interview session

III.Each candidate will appear at least once

IV.If it becomes necessary to call applicants for additonal interviews, no more 1 such applicant should

be asked to appear the next week

V.Because of a detail in the written applications,it was agreed that whenever candidate B appears, A

should also be present.

VI.Because of travel difficulties it was agreed that C will appear for only 1 interview.

1.At the first interview the following candidates appear A,B,D.Which of the follwing combinations can

be called for the interview to be held next week.

A.BCD

B.CDE

C.ABE

D.ABC

Ans.B

2.Which of the following is a possible sequence of combinations for interviews in 2 successive weeks

A.ABC;BDE

B.ABD;ABE

C.ADE;ABC

D.BDE;ACD

Ans.C

3.If A ,B and D appear for the interview and D is called for additional interview the following

week,which 2 candidates may be asked to appear with D?

I. A

Page 45: 7116273 Aptitude IGATE Infosys Tcs Ect

II B

III.C

IV.E

A.I and II

B.I and III only

C.II and III only

D.III and IV only

Ans.D

4.Which of the following correctly state(s) the procedure followed by the search committee

I.After the second interview all applicants have appeared at least once

II.The committee sees each applicant a second time

III.If a third session,it is possible for all applicants to appear at least twice

A.I only

B.II only

C.III only

D.Both I and II

Ans.A

3.  A certain city is served by subway lines A,B and C and numbers 1,2 and 3.

When it snows , morning service on B is delayed

When it rains or snows , service on A, 2 and 3 are delayed both in the morning and afternoon

When temp. falls below 30 degrees farenheit afternoon service is cancelled in either the A line or the

3 line,

but not both.

When the temperature rises over 90 degrees farenheit, the afternoon service is cancelled in either

the line C or the

3 line but not both.When the service on the A line is delayed or cancelled, service on the C line which

connects the A line, is delayed.When service on the 3 line is cancelled, service on the B line which

connects the 3 line is delayed.

1. On Jan 10th, with the temperature at 15 degree farenheit, it snows all day. On how many lines will

service be

affected, including both morning and afternoon.

(A) 2

(B) 3

(C) 4

(D) 5

Ans. D

2. On Aug 15th with the temperature at 97 degrees farenheit it begins to rain at 1 PM. What is the

minimum number

of lines on which service will be affected?

(A) 2

Page 46: 7116273 Aptitude IGATE Infosys Tcs Ect

(B) 3

(C) 4

(D) 5

Ans. C

3. On which of the following occasions would service be on the greatest number of lines disrupted.

(A) A snowy afternoon with the temperature at 45 degree farenheit

(B) A snowy morning with the temperature at 45 degree farenheit

(C) A rainy afternoon with the temperature at 45 degree farenheit

(D) A rainy afternoon with the temperature at 95 degree farenheit

Ans. B

4. In a certain society, there are two marriage groups, red and brown. No marriage is permitted within a

group. On marriage, males become part of their wives groups; women remain in their own group.

Children belong to the same group as their parents. Widowers and divorced males revert to the

group of their birth. Marriage to more than one person at the same time and marriage to a direct

descendant are forbidden

1. A brown female could have had

I. A grandfather born Red

II. A grandmother born Red

III Two grandfathers born Brown

(A) I only

(B) III only

(C) I, II and III

(D) I and II only

Ans. D

2. A male born into the brown group may have

(A) An uncle in either group

(B) A brown daughter

(C) A brown son

(D) A son-in-law born into red group

Ans. A

3. Which of the following is not permitted under the rules as stated.

(A) A brown male marrying his father's sister

(B) A red female marrying her mother's brother

(C) A widower marrying his wife's sister

(D) A widow marrying her divorced daughter's ex-husband

Ans. B

Page 47: 7116273 Aptitude IGATE Infosys Tcs Ect

4. If widowers and divorced males retained their group they had upon marrying which of the

following would be permissible ( Assume that no previous marriage occurred)

(A) A woman marrying her dead sister's husband

(B) A woman marrying her divorced daughter's ex-husband

(C) A widower marrying his brother's daughter

(D) A woman marrying her mother's brother who is a widower.

Ans. D

5. I. All G's are H's

II. All G's are J's or K's

III All J's and K's are G's

IV All L's are K's

V All N's are M's

VI No M's are G's

Q1. If no P's are K's which of the following must be true

(A) No P is a G

(B) No P is an H

(C) If any P is an H it is a G

(D) If any P is a G it is a J

Ans. D

2. Which of the following can be logically deduced from the stated conditions

(A) No M's are H's

(B) No H's are M's

(C) Some M's are H's

(D) No N's are G's

Ans. D

3. Which of the following is inconsistent with one or more conditions

(A) All H's are G's

(B) All H's are M's

(C) Some H's are both M's and G's

(D) No M's are H's

Ans. C

4. The statement "No L's are J's" is

I. Logically deducible from the conditions stated

II Consistent with but not deducible from the conditions stated

III. Deducible from the stated conditions together with the additional statements "No J's are K's"

(A) I only

(B) II only

Page 48: 7116273 Aptitude IGATE Infosys Tcs Ect

(C) III only

(D) II and III only

Ans. D

Technical

1. What is meant by polymorphism

Ans. Redfinition of a base class method in a derived class

2. What is the essential feature of inheritance

Ans. All properties of existing class are derived.

3. What does the protocol FTP do

Ans. Transfer a file b/w stations with user authentification

4. In the transport layer ,TCP is what type of protocol

Ans. Connection oriented.

5. Why is a gateway used

Ans. To connect incompatible networks.

6. How is linked list implemented

Ans. By referential structures.

7. What method is used in Win95 in multitasking?

Ans. Non preemptive check.

8. What is meant by functional dependency?

9. What is a semaphore?

Ans. A method synchronization of multiple processes

10. What is the precedence order from high to low ,of the symbols ( ) ++ /

Ans.( ) , ++, /

11. Preorder of A*(B+C)/D-G

Ans.*+ABC/-DG

12.  B-tree (failure nodes at same level)

13.  Dense index (index record appers for every search -key in file)

Page 49: 7116273 Aptitude IGATE Infosys Tcs Ect

14. What is the efficiency of merge sort

Ans. O(n log n)

15. A program on swaping ( 10,5 )was given

16. In which layer are routers used

17. Ans.In network layer

18. In which layer are packets formed ( in network layer )

19. heap ( priority queue )

20. Copy constructor ( constant reference )

21. 26.Which of the following sorting algorithem has average sorting behavior --

Bubble sort,merge sort,heap sort,exchange sort

Ans. Heap sort

22. In binary search tree which traversal is used for getting ascending order values--Inorder ,post

order,preorder

Ans.Inorder

23. What are device drivers used for

Ans.To provide software for enabling the hardware

24. Irrevalent to unix command ( getty)

25. What is fork command in unix

Ans. System call used to create process

26. What is make command in unix

Ans. Used forcreation of more than one file

27. In unix .profile contains

Ans. Start up program

28. In unix echo is used for ( answer C)

29. In unix 'ls 'stores contents in

Ans.inode block

Celstream

On the basis of a Campus  Interview conducted on 04-sep-2003 (PESIT Bangalore)

Page 50: 7116273 Aptitude IGATE Infosys Tcs Ect

   Pattern:

   No negative marking. 

  Section1: 20 aptitude

  Time 15 minutes.

1. One problem like 5pigs cost is same as 25 dogs which is equal 12 elephants if 15  elephants

is 2500 what is value of 1 pig. (numbers are not same.)                                                     

2. One man brought pen and book from 250 and he sold both at 10% profit on pen and 20%

loss on book and finally he gained 40% what is the value of pen (numbers are not same)

3. 12 men can build wall of length of 12m in 10 days how many men are required to build 20 m

in 15 days. (numbers are not exact)  answering of at least 9 is necessary. try to answer the questions

which are easy. do some guessing work otherwise u can't complete the paper

4. Passage like this  there are 2 wings of houses east and west u can see the sea from east

from some houses for that extra charges. and some rooms had kitchen extra cost for them and so

on  and questions on that 

5. Passage like this  there are 2 wings of houses east and west u can see the sea from east

from some houses for that extra charges. and some rooms had kitchen extra cost for them and so

on  and questions on that 

6. In a temple there are 3 doors. and after each doar there is one idol (so there will be 3 idols)

at each door the flowers we take will be doubled . and questions on this answers were for one 7 and

for another it is 40. and one more question was there i don't no that

7. One passage was on playing cards. know how many diamonds and etc.            

8. There was on passage like this one host and one hostess. invited 6 for lunch and they are

sitting in on a rectangular table for their lunch. and arrangements should that  at least one adjacent

person should be apposite sex. and some conditions are given easy to solve. for one question the

answer was host.

ChangepondThere were two sections Aptitude Technical

Aptitude

1. If m-4 is 7 more than n then m+9 is how much more than n+2

2. If 10x coins are added to the no. of original coins he has (5y + 1) times more

coins. Find out how many coins he 

had originally in terms of x and y 

3. Find out the Simple Interest paid for a sum of $4000 at the rate of 8% per annum

for 3 months.

4. If it takes 10 technicians working 6 hours to build a server. They start woring at 11

AM and 1 technician is

 added  per hour starting at 5 PM. At what time they will finish the server?

5.  A chart was there giving the speed of train after some minutes like

  0 30 60 90 120 150 180

  40 45 50 55 60 65 70

Page 51: 7116273 Aptitude IGATE Infosys Tcs Ect

6. Find the % increase in the speed of train?

7.  If it is extrapolated further what will be the speed after 5 hrs.

8. If the data is for 10 bogies and for every 2 bogies added speed decreases by 5%

what will be the speed after 5 

hrs.

9.  The type of questions where Both stmt are necessary First one is sufficient in

itself…..… What is the perimeter of the quadrilateral?

10. The quadrilateral is rectangle

11. The area of the quadrilateral is 36 What is a*b

12. 6a+4b=43

13. 3a+12b=63

14. The house is big and lovely but since nobody uses it it is pretty much a white

Elephant What does White     Elephant  mean?

15. Sometime they irritate me because they are up with the lark whereas I like to lie in

What does Up with Lark 

mean?

Technical

1. What exception can an overridden method throw in comparison with the method it

is overriding?

2. What does parseQueryString return?

3. Anonymous classes – Can they have constructors?

4.  what is wrong with this

abstract class MyClass

{

transient a,

synchronized b.

}

5. How can an inner class access the members of outer class?

6. Find the output:

String a=”abc”;

String b=”abc”; 

If(a==b)

System.out.println(“1”);

Else

System.out.println(“2”);

Page 52: 7116273 Aptitude IGATE Infosys Tcs Ect

If a.equals(b) 

System.out.println(“3”);

Else

System.out.println(“4”);

7. Find the output Code showing Call by reference and call by value was given Array

was used.

8. Some For stmts was given Mark all valid once

9.  byte b=1

While(++b > 0);

System.out.println(b);

Deloittee

Recruitment test of  Deloitte Tech at Coimbatore on  30. 07.2004

There are three sections (Verbal, Quans and Critical)15 Quans10 Verbal4 (Fitting the correct word in the given sentence)6 (Fitting the correct sentence in the given paragraph)15 Reasoning.

1. I have 20 rupees. I bought 1, 2, 5 rupee stamps. They are different in numbers by the reason

of no change, the shop keeper gives 3 one rupee stamps. So how many stamp(s) I have. Ans: 10

2. An Engine length 1000 m moving at 10 m/s. A bird is flying from engine to end with x kmph

and coming back at 2x. Take total time of bird traveling as 187.5 s. Find x and 2x.

3. All persons know either swimming or rowing. There are 14 persons who know only swimming

and 14 persons who know only rowing. 8 tickets sold for rowing. How many people are there for

swimming?

4. Which polygon has no. of sides = diagonal (Eg. Pentagon)

5. One Cigar can be made from 7 bits. If there are 16 Cigars then how many bits collected?

Ans: 4

6. A, B, C, D went to a hotel and planned to share the bill equally. But afterwards they changed

their plan and to pay proportional to consumption A paid 240, B & C paid equally, D paid only half the

amount that he should have paid based on the first plan. What is the amount paid by B?

7. There is a point P on the circle. A and B started running in two constant different speeds. A in

Clockwise and B in Anti-clockwise. First time 500 m in Clockwise from P then 400 Anti-clockwise. If B

is yet to complete one round, What is the circumference of the circle?

8. There are 5 Sub with equal high marks. Mark scored by a boy is 3:5:6:7:8 (Not sure). If his

total aggregate if 3/5 of the total of the highest score, in how many subjects has he got more than

60%?

9. There are 11 lines in plane. How many intersections (Maximum) can be made?

10. There are 3 Sections with 5 Qns each. If three Qns are selected from each section, the

chance of getting different Qns is________

11. There is a 20 X 20 array. In Each row , the tallest person is called and among them, the

tallest person is A. In Each column, the shortest person is called and among them, the shortest

person is B. Who is taller?

Page 53: 7116273 Aptitude IGATE Infosys Tcs Ect

12. P # Q = (P-Q)(Q-P) = - 1. Then Which is true?

P = 3, Q = 2 P = 2,Q = 3 P = -1,Q = 1 P = 1, Q = -1 Ans: I & II only

13. 7 Pink, 5 Black, 11 Yellow balls are there. Minimum no. atleast to get one black and yellow

ball Ans: 17

14. A, B, C and D are four people. There are four houses Red, Yellow, Blue, White. P, Q, R and S

are four sections not in same order Conditions like  Three are sisters B comes from Red C comes

from Blue Qns were asked based on that

15. A Father is willing his estates like this. If a boy is born, wife has 1/3 part and remaining for

boy. If a girl is born, Wife has 2/3 and remaining for the girl. But twins (Boy + Girl) are born. What is

the share that the daughter would get?

16. MBA, GRE prob from Barrons GRE (Don know whether it is there in all GRE Editions). Ans:

GMAT, CAT (Sure)

17.  If ü - Married 

  û - Not Married and

  M-ü N-û

  N-ü L-û

  L-û M-ü

Who is married? Ans: N

 Interview

  (A panel with 2 people- one HR & one Tech Guy)

18. Tell me abt urself.

19. You specify that you get irritated when things dont go in the pace you want them to.Then

how do you handle a team?

20. tell us about ur projects

21.  Tell me wht is a foreign key(DBMS) .First tell me what is a key?

22. What happens when an application is opened as like an MS Word and u press Tab key then

the response seen

is the cursor moving a bit on the Word document.So,what r the processes (no jargon,give the proper

technical terms) behind this whole thing happening? 

23. Do u think ur project is saleable? What rating

24. What r the things u will do to reduce the processing time of your project?

25. What can you do to decrease the time taken for processing a file in a batch processing

system?

DSL SoftwareDeutsche - Placement sample question papers 

1. What is the percentage represented by 0.03 * 0.05 ?

(a)0.0015

(b)0.000015

(c)0.15

(d)15

Ans.B

2. (x-a)(x-b)(x-c)....(x-z) = ?

(a) 1

(b) -1

(c) 0

(d) Can't be determined

Page 54: 7116273 Aptitude IGATE Infosys Tcs Ect

Ans. C

3. If a = 1, b = 2, c = 3.......z = 26 what is the value of p+q+r ?

(a)33

(b)51

(c)52

(d)48

Ans. B

4. A is 8 miles east of B.

C is 10 miles north of B. 

D is 13 miles east of C and E is 2 miles north of D.

Find shortest distance between A and E. 

(a) 5 miles

(b) 6miles

(c) 13 miles

(d) 18 miles

Ans. C

5. If z = 1, y = 2.......a = 26. Find the value of z + y + x + .......+a.

(a) 351

(b) 221

(c) 400

(d) 200

Ans. A

6. There are 30 socks in a bag. Out of these 60 % are green and the rest are blue. What is the maximum

number of times that socks have to be taken out so that atleast 1 blue pair is found. 

(a) 21

(b) 2

(c)18

(d) 20

Ans. D

7. How many two digit numbers have their square ending with 8.

(a) 13

(b) 12

(c) 0

(d) 11

Ans. C

8. How many numbers are there between 100 and 300 with 2 in the end and 2 in the beginning.

(a) 10

(b) 9

(c) 11

(d) none of these

Ans. A

9. 0.000006 * 0.0000007 = ?

(a) 0.0000000042

(b) 0.000000000042

(c) 0.0000000000042

Page 55: 7116273 Aptitude IGATE Infosys Tcs Ect

(d) 0.00000000000042

Ans. B

10. You have Rs 1000 with 8% p.a compounded every 6 months. What is the total interest you get after 1

year.

(a) Rs.116.40

(b) Rs.345.60

(c) Rs.224.50

(d) Rs.160

Ans. A

11. If x + y =12,

x - y = 2

Find x + 2y.

(a) 12

(b) 17

(c) 14

(d) none of these

Ans. B

12. With one gallon of petrol a person moves at a speed of 50 mph and covers 16 miles. 3/4th of the distance

is covered while moving at 60 mph. How many gallons does he need to cover 120 miles in 60 mph.

13. A tap drains at x speed while tap B is closed. When both taps are open they drain at y speed. What is the

speed of draining when only tap B is open

(a) x - y

(b) y-x

(c) x

(d) can't be determined

Ans. B

14. What is twenty percent of 25 % of 20.

(a) 2

(b)1

(c) 5

(d) 4

Ans. B

15. A rectangle has the dimensions 6ft * 4ft. How many squares of 0.5 inches will it need to completely fill it.

(a) 32000

(b) 12824

(c) 13824

(d) 18324

Ans. C

Directions for questions 16-21: In each question,a series of letters satisfying a certain pattern are given.

Identify the pattern and then find the letter/letters that will come in place of the blank/blanks.

16. a, c, e, g, _

(a) h

(b) i

(c) d

(d) j

Page 56: 7116273 Aptitude IGATE Infosys Tcs Ect

Ans. B

17. a, e, i, m, q, u, _, _

(a) y, c

(b) b, f

(c) g, i

(d) none

Ans. A

18. ay , bz , cw , dx ,__

(a) gu

(b) ev

(c) fv

(d) eu

Ans. D

19. 1, 2, 3, 5, 7, 11, __

(a) 15

(b) 9

(c) 13

(d) 12

Ans. 13 , series of prime numbers

20. kp , lo , mn , __

(a) nm

(b) np

(c) op

(d) pq

Ans. A

21. abc , zyx , def , wvu , ___

(a) ghi

(b) tsr

(c) ihg

(d) str

Ans. A

22. How is my mother's sister's brother's wife's child related to me?

(a) brother

(b) uncle

(c) cousin

(d) nephew

Ans. A

23.What will my mother's husband's father-in-law's son's daughter to me?

(a) niece 

(b) aunt

(c) sister

(d) none of these

Ans. D

24. A person travels 12 km in the southward direction and then travels 5 km to the right.  He then travels 5

Page 57: 7116273 Aptitude IGATE Infosys Tcs Ect

km to the right and then travels 15 km towards the right and finally travels 5km towards the east. How far is

he from his starting place?

(a) 5km

(b) 3m

(c) 3.5km

(d) 4km

Ans. (b)

25. A person travels towards east from his house and travels a distance of 3 meters. He then ran a distance

of 7 meters southwards and then travels towards east a distance of 3 meter Finally he travels southwards a

distance of 10 meters.What is his vertical distance from his house?

(a) 10

(b) 17

(c) 12

(d) none of these

Ans. B

26. The sum of three consecutive numbers is 132. Find the square of the largest number.

(a) 2026

(b) 640

(c) 900

(d) 2025

Ans. D

27. Father's age is five times his son's age. Four years back father was 9 times older than his son.Find their

present ages

(a) 30yrs, 6yrs

(b) 25yrs, 5yrs

(c) 40yrs, 8yrs

(d) none of these

Ans. C

28. If x + 5y = 6 ;

7x - 2y = 5

Find x - y

(a) 1

(b) 2

(c) 0

(d) none of these

Ans. C

29. If the equation x2 + 2(k+2)x + 9k = 0 has equal roots, find k?

(a) 1,4

(b) 1,5

(c) 2,5

(d) can't be determined

Ans. A

Directions for questions 30 to 35: Select the alternative that logically follows form the two given statements.

30. All books are pages. All pages are boxes.

(a) All boxes are books

(b) All books are boxes

Page 58: 7116273 Aptitude IGATE Infosys Tcs Ect

(c) No books are boxes

(d) Both (a) and (b) are correct

Ans. B

31. No apple is an orange. All bananas are oranges.

(a) All apples are oranges

(b) Some apples are oranges 

(c) No apple is a banana

(d) None of the above

Ans. A

32. All pens are elephants. Some elephants are cats.

(a) Some pens are cats 

(b) No pens are cats

(c) All pens are cats

(d) None of the above

Ans. D

33. All shares are debentures.No debentures are deposits.

(a) All shares are deposits 

(b) Some shares are deposits

(c) No shares are deposits 

(d) None of the above

Ans.C

34. Many fathers are brothers. All brothers are priests.

(a) No father is a priest

(b) Many fathers are not priests

(c) Many fathers are priests

(d) Both (b) and (c)

Ans. B

35. Some pens are pencils. All pencils are costly.

(a) No pens are costly

(b) Some pens are costly

(c) All pens are costly

(d) None of the above

Ans. B

36. How many two digit numbers have their squares ending with 6?

(a) 9

(b) 10

(c) 164

(d) 18

Ans. D

37. In how many years will $1200 amount to $1323 at 5%p.a. compound interest?

(a) 3 years

(b) 2 1/2 yrs

(c) 2 yrs

(d) none of the above

Ans. C

Page 59: 7116273 Aptitude IGATE Infosys Tcs Ect

38. How many terms of the series -9 , -6 , -3 ,.........must be taken such that the sum may be 66?

(a) 11

(b) 13

(c) 9

(d) 10

Ans. A

39. The sum of three numbers in AP is 27 and their product is 504. Find the terms.

(a) 7, 9 ,11

(b) 5, 9, 13

(c) 4, 9, 14

(d) none of these

Ans. C

40. Two pipes A and B can fill a tank in 20 and 25 seconds respectively.  Both are opened at the same time

and after 5 minutes the first one is closed.How much more time will it take for the tank to be full?

(a) 13 min 45 secs

(b) 10 min

(c) 18 min

(d) 15 min 45 secs

Ans. A

41. How long will a train 150 metres long and travelling at a speed of 45 kmph, take to cross a platform of

length 250 meters?

(a) 48 secs

(b) 32 secs

(c) 30 secs

(d) none of the above

Ans. B

42.Walking at 4/5th of his usual speed, a man reaches his office 15 mins late. Find his usual time for reaching

the office?

(a) 45 mins

(b) 2 hrs

(c) 1 hr

(d) 30 mins

Ans. C

43. If the word "CODING" is represented as DPEJOH , then the word "CURFEW" can be represented?

(a) DVSGFX

(b) DVSHFX

(c) DGSHFX

(d) DTSGFY

Ans. A

44. If in a certain code "RANGE" is coded as 12345 and "RANDOM" is coded as 123678, then the code for the

word "MANGO" would be 

(a) 82357

(b) 84563

(c) 82346

Page 60: 7116273 Aptitude IGATE Infosys Tcs Ect

(d) 82543

Ans. D

45. If in a code "COPIER" is coded as 1234 and "REAM" is coded as 5421,then "DREAM" is coded as 

(a) 35421

(b) 43512

(c) 35412

(d) 34521

Ans. A

Directions for questions 46-48:The questions are based on the following data In a class of 150 students 55

speak English;85 speak Telugu and 30 speak neither English nor Telugu

46. How many speak both English and Telugu?

(a) 10

(b) 15

(c) 20

(d) 12

Ans. C

47.How many speak only Telugu?

(a) 55

(b) 45

(c) 65

(d) none of the above

Ans.C

48.How many speak at least one of the two languages?

(a) 120

(b) 100 

(c) 250

(d) 50

Ans. A

49. Find the lowest of the three numbers in a GP whose sum is 38 and product 945.

(a) 12

(b) 18

(c) 4

(d) 8

Ans. D

50. A can do a piece of work in 24days. When he had worked for 4 days B joined him. If the complete work is

finished in 16 days form the beginning, in how many days can B alone finish the work?

(a) 24 days

(b) 18 days

(c) 36 days

(d) None of the above

Ans. C

DSQ

Page 61: 7116273 Aptitude IGATE Infosys Tcs Ect

DSQ PAPER. - Aug-04

Technical aptitude test

1. the 2's complement number of 110010 isans; none of those2.Truth table of a logic functionans: displays all its input output possibilities3.The process of varying one signal according to the patternprovided by another signal isans:modulation4.The octal equivalant of hexa number 123 isans:443.Determine how many times the given loop is executed5.m=3 while (m&lt;18) do {m=m+2;m=m-1}ans:156.m=1;a=30;while (m&lt;&gt;1) do {m=m+1;a=a-1;}7.m=1;a=30;while (e*m-5&lt;a) do {m=m+1;a=a-2}ans:9in questions 8 to 12 find the values of a and b at the end of thexecution of the program segment8.a=2;b=1;x=1;while(x&lt;=4)do {a=a+b;x=x+1}ans:(5,1)9.a=1;b=2;x=1 while (x&lt;=5)do {b=a+b;x=x+1;}ans:(1,6)10.a=0;b=0;x=1;while (x&lt;=4){a=a+x;b=b-x;x=x+1;}ans:(10,-10)In questions 11-15 find the odd man out11.address12.cmos13.lisp ( from 11-15 these are the answers dont confuse)14.mouse15.csh16.The complexity of bublle sort is0(a),then kequalsans:217.In BCD,the decimal number 516 isans:1010111018. RS 232 is standards for ans:physical layer19. which of the following is not true of a DBMSans:maintain integrity20.the method of access used for obtaining a record from a tape isans:sequential21.Whichof the following is an universal gateans:nor22.an ASCII text file containing 500 characters will have size approximatelyans:500bytes23.which of the following is a vectorans;force24.The lowest attainable temperatureans 0k25.viscosity is most similar toans:friciton26.The elapsed time between the submission of the job and gettingthe output isans:seek time27.the earliest calculating device still in use isans: abacus28.an acronym for the organisation that publishes programminglangauge standards is ans:ANSI29.Qunatities used to bring fields to standards sixe areans:pixels30.the base of a number system is called asans:radix31.An 8kb computer will bave addresses -----ans:819132.Arranging data in a specific form is calledans: sorting33.A translator which reads an entire programme written in a highlevel language and converts it into machine language code isans: interpreter34.a data hierarchy in assending order isans:bit-byte -field-record-file-database35.a computer generated output that lets programmer follow the executionof the program line by line is 36.data items grouped togother for storage form a 37. the most dangerous aspect of computer virusesis their ability to

Page 62: 7116273 Aptitude IGATE Infosys Tcs Ect

ans: change system memory38.a distributed data processing configuration in which allactivities must pass through an centrally located computer is called aans: ring network39.communication circuite that transmit data inboth directions butnot at the same time areoperating in40.which of the following is not an application softwareans:unix41.For which of the folowing computer application is real timeprocessing not essentialans;42.a data structure with one to many relationship is a ans:tree43. which of the following hardware components is most important to operationof a database management systemans:high speed,large capacity disk44. computer viruses can spread from one system to another by means ofans:all the above45. afront end processor is ususally used inans: multi processing46.a radioactive material of mass 16 grms in 10 years due toradiation. how many years will it take for the material to attain a mass f1 gmans:3047.a block of ice floats on water in a beaker, as the ice melts,the waterlevel in the beaker willans:remain same48.if va,vn,vs are velocities of sound in air,water, and steel then,ans:vs&gt;vn&gt;va49.in usual computer arthemetic, the value of the integer expression22/5*2+8*2/6ans:1050.an operating system is

ans: all the above

REASONING ABILITY TEST (VERBAL AND GENERAL)

Questions 1to 10 are odd man out i am giving only answers you just remember

1.sphere2.rhine3.s4.5485.voluminous6.graphologist7.8.nepal9.egypt10.squash

11. find the next letter in the series B,F,J,P ---ANS:V12. find the next letter in the series O,T,T,F,F,S,S--ANS:E13. find the next letter inthe series Q,W,E,R,T,--ANS:B14. FIND THE NEXT LETTER IN THE SERIES c,o,m,p,u,t,e--ans:r15. find the next letter in the series A,C,F,J,0--U16. FIND THE NEXT letter in the series C,O,M,P,U,T,E,---ANS:R

17. FIND the next number in the series 11,112,1124,11248,--ans:112481618.find the next number in the series 12,21,,112,121,122---ans:22119.find the nuxt number in the series 1,2,5,10,,15,22--ans:2920.find the next number in the series 0,3,8,15,,24,48,63--ans:80

QUESTIONS 21 TO 35 FILL IN THE BLANKS USING THE WORDS GIVEN 1 TO 4 AND

P TO S CHOOSE THE APPROPRAITE RESPONSE FROM A TO D.

21. -------is to district as district is to-------

1)country 2)continent 3)state 4)villagep)town q)village r)state s)cityans)3-P

Page 63: 7116273 Aptitude IGATE Infosys Tcs Ect

22. -------is to seeds as hen is to ----------1)embrayo 2)plant c)cock d)chickenp)bird q)craaper r)egg s)treeans) 2-R

FROM HERE ON WARDS I AM GIVING QUESTION TO ANSWER23. ------is to animal as ----is to plantsans)blood-sap (4-S)

24. ------is to cardic as brain is to ------------ans)heart-nerves (2-P)25. --------is to myopia as long sight is to -------ans)26) ----- is to japan as rupee is to -----ans)YEN -INDIA (3-P)27) ------is to christian as synage gue is to ------ans)church-Jaws28) -----is to thermometer as velocity is to ------ans)Temperature-animeter29) ------is to sentence as act is to-------ans)paragraph -scene30) ------ IS TO EARTH AS EARTH IS TO ----ANS)Moon -sun31) -------is to london as yamuna is to ------ans)thames-delhi32) -----is to india as alps is to-----ans)himalayas -switzerland 33) ------ is to proton as cathode is to------ ans)electron -anode34) ------is to a river isto branch is to --------ans)lake-tree35) ------is to proofread as account is to -------ans)proof -audit

MATHEMATICS TEST

1. if a**2+b**2+c**2=1 then ab+bc+cd lies in the intrval

a. -1/2,1 b.-1,1 c.-1/2,2 d.-1,22.if x is root of 4*y**2+2*y-1=0, the its other root is given bya.4x**2+2*x+1 b.x-1/2 c 2x-1 d.2-x4. the perimetere of a square is 44m more than that of another and187 sqm more in area then the side of the larger square isans:17m5. the value of x which makes xi+2j+3k and -i+5j+k perpendicular isans:none of the above6. the value of determiterment ---|matrix|ans:67. the solution of the system of equations 2y-z=0,x+3y=-4,3x+4y=3 isans: x=5,y=3,z=68. the radius of the circle 4x**2+4y**2 =100 ans: 59. the ages of two brothers are now in the ratio 4:3 but fifteenyears ago, they were in the ratio 3:2the present age areans:4510.if z is the arithemtic mean between x and y , then the value of x/(x-z)+y/(y-z)is ans:211.the least integer n for which the sum 1+3+5+---+(2n+1) exceeds9999 is ans:10012. the number of subsets of a set s is 64, then s hasa: 8 b.6 c.4.d. 16 elements13.if n is a natural numbers, the n(n+1)(N+5) ISans:a multiple of 614.a spere circumscribes a cylinder . then the ratio of the surfacearea of the spere to the curved surface area of the cylinder isans:3:215.ax/b+b=bx/a+a,a//b. then x equalsans:ab/(a+b)16. if logs sqrt(5x)=1,then x equalsans:17. which of the following triplets cannot be sides o a ritht angledtriangleans: (p-q),(p+q),sqrt(p+q)18.inwhat time will RS. 3200 amount to RS.3528 at 5% annum compound interest

Page 64: 7116273 Aptitude IGATE Infosys Tcs Ect

ans:2 years19.if the price of orange falls by 20 % one can buy 5 dozens morefor RS 300, THEN THE ORIGINAL price per dozen is ans:1520. the triangle formed by (0,0),(0,4,),(3,8) isans: right angled triangle21. The area of parallegram (0,0),(0,4),(3,8) isa.12 b.6 c.24 d.2522. A man can do a piece of work in 9 days, a woman can do 3/4 asmuch work as a man in one day and a boy in twice the time that a mantakes. How many days wil 2 men, 3 women and 5 boys together take to do thework?ans:4/3 days23.The probability of getting at least one head in a single throw ofthree coins isans:7/8 24.A problem is given to two student A and B WHOSE REspectivechances of solving it are 1/2 and 2/3 . IF BOTH OF THEM TRY TO SOLVE THEPROBLEM INDEPENDENTLY, THE PROBability that the rpoblem will be solved isans:5/625.In a single throw of 3 dice, the chance of throwing a total of 15isans:a.1/15b.5/216 c.1/3 d. 1/426.(3x-2)/(x-2)&lt;=0,thenans:2/3&lt;=x&lt;240 . P and Q are positive integers with their averagfe 15, find howmany different values can one takeans:2941.The hands of clock coincide after every 66 min of correct time.How much does the clock gain or lose in a day.ans:gains 11 min42.Ifxto the power of 5=4 and x to the power of 4=5/y , then x equalsans:4y/543.if x,y,z are natural numbers, and if 2x=y and y=3z, which of thefollowing numbers could be x+y+zans:4444.The pipes can fill a cistern in 1.5 hours and 2 hoursrespectively. A wste pipe can empty a full cistern in 3 hours. If thecisternis empty and all three pipes are opened together, in how many minwill the cistern be half fullans:3645. The members a,b,12 forma geometric progression and the nos a,b,9form an arithmetic progression. find the value of a+bans:9or 4546.IN a group of 15,7 can speak spanish, 8 can speak french, and 3can speak neither.How much of the group can speak both french andspanishans:2/347.Oranges are bought at 11 for Rs. 10 and sold at 10. for Rs. 11the profit isans:21%48.If x/y=2/3 then y**2/x**2 is equal toans:9/449. the cordinates of A,b,c are respectively (2,3) ,(4,4) AND(0,-2). If abcd is a parallelgram. The coordinates of D areans:(-1,-3)50. If nis odd which of the following statements is trueans: 1.n is odd 2. n**2 is odd 3. n**2 is evena.1 only b.2 only c. 3 only 4. 1&amp;2 only

Efunds

efunds Sample Test Paper

1. Aptitude test - 35 aptitude questions & 15 engineering questions (1 hr) There is some min cut off Book of R

S Agarwal is enough 4 aptitude test Eng also easy..... synonyms, sentence completion

2. Computer awareness test - 25 questions(15 min) questions from C,OS,DBMS,n/w,data structures.... some

questions

1) Deadlock involves

a) hold & wait b) circular chain c) .... d) all of these

Page 65: 7116273 Aptitude IGATE Infosys Tcs Ect

Ans) d

2) Some questions related 2 windows.... Ans) FAT 32 

3) Which of these is not a feature of AVL tree

a) Balanced b) while insertion its rotated... etc

4) Feature of primary key

a) clustered b) nonclustered c) unique but clustered or no clustered... etc

5) Which is true 4 referential integrity

a) while insertion in foreign key its master table is checked

b) while insertion in master table its foreign key is checked

c) while deletion of foreign key its master table is checked.....etc

6) Given that some 110101 * xx = 1001101 ( data mite b wrong ) find the value of x 

( related 2 binary xion)

7) Virtual mem concept involves

a) lazy swappin.... etc

8) Which of these is not a windows system file.... some choices

  

3. Technical interview (30 - 45 min) questions from C, C++, Area of interest & final sem project

if v study "pointers in C" by "Y.Kanetkar" then v can

answer C

questions.other qns r general concepts... b confident while answering questions

  

4. HR interview (10 - 15 min) .... some questions..

1) Tell about yourself & your family background?

2) your strength, weakness?

3) which is important 4 a s/w - development or testing?

4) what kind of work enviornment do u prefer?

5) what do u know about our company?

Epson

Sample Questions

1. 6 persons A,B,C,D,E,F are playing a game of cards . A's father and mother , uncle were in

the gang. There were two women. B is the mother of A gets morepoints then her husband. D got

more points then E but less then F. Niece of E got the lowest points. Father of A agot More points

then F. But would not win the game.

1. who won the game? 

a)A b) B c) F d) D ans: b

2. who got lowest points ?

a) A b) C c) E d) B ans: a

3. who is the husband of B ?

a) F b) E c) D d) c ans : c

4. B was one of the lady. who was other lady?

a) C b) D c) E d) A ans: a

5.  who stood second in the game?

a) A b) B c) C d) D ans: c

2.  A is the widow of B. B & C were the only children of E. C is unmarried and is a doctor. D is

the grand daughter of E and studies science. How is A related to D.

Page 66: 7116273 Aptitude IGATE Infosys Tcs Ect

a)Aunt b) daughter c) sister d) sister_in_law ans: b

Follow the following for Q. 3-4.

Only conclusion I follows definately.

b) ,, II ,,

c) Both I & II follow

d) Neither I nor II follows.

e) either I or II follows.

3. STATEMENTS:

It is some times said that corruption is due to proverty and gross diparities.

It is an uncalled for slurr on the poor. Because the poor in India are among the most unsight

persons.

conclusions: 

I. upsight persons can't be corrupt persons.

II. Disparities of income make the people corrupt.

ans: c

4. TATEMENTS:

The Indian economy has suffered considerble stresses and strains during the lostthree years. The

general picture is one of stagnations. 

conclusions: 

I. stresses and strains in economy lead to stagnation . 

ð 7 3 

ŠII. Indian economy is stagnant.

ans: e

5-12 follow this answers.

a) only inference I follows.

b) only inference II follows.

c) Both I & II follow.

d) neither I nor II follows.

5. no hawk is a sparrow.

All hawks are hens.

Inferences:

I. no hen is a sparrow.

II. no sparrow is a hen.

ans: c

6. all graduates are chairs.

all chairs are tables.

Inferences:

I. all graduates are tables.

II. all tables are graduates.

ans: a

7. some birds are elephnts.

some elephants are white.

Inferences:

I. some birds are white

II. some white are birds

ans: d

Page 67: 7116273 Aptitude IGATE Infosys Tcs Ect

8. all fans are vans.

none van is pan.

I. No pan is fan.

II.no fan is pan.

ans: d

9.  all boats are coats.

no coat is shirt.

I. all boats are shirts

II. all shitrs are boats.

ans: a

10. no pen is hen.

some hens are healthy.

I. no pen is healthy.

II. no healthy thing is pen.

ans: d

11. all hopes are doops.

all fears are hairs.

Below we are giving Q'es whatever remember to us. It is not clear and order.

12.  poem, song, rommance, dirty,couplet

ans: Rommance

13. Historical, ancient, olden ,inheritant.

ans: inheritant.

14. widow ..........., ans: daughter.

15. For one coding problem ( ie he will give some numbering to alphbets )

ans: 

1) PRIQOV

2) MOFZB

16. synonyms:

i) placeble ii) emblish iii) incence iv) depricate v) palret

17. ODDMEN OUT: 

i) bench, chair, winch,thrown,pedastal ans: pedestal

ii) book, volume, saga......

iii) 3,4,20,38 ....... ans: 20

iv) 3,6,52.5,157.5 ....ans: 52.5

18. one age problem father and son 10 years ago........ans: 14,38.

Future

Future Soft Sample Test Paper

1.  S --> AB|AS

 A --> a|aA

 B --> b

What is the grammer accepted by the above?

Ans. aa*b

Page 68: 7116273 Aptitude IGATE Infosys Tcs Ect

2. How many address lines are needed to address a 64Kb segment with each register storing

upto 512 bytes.

Ans. 14 address lines

3.  Find the expression representing the following K-map

1   1 1

       

  1 1  

1   1 1

4.  For the POS form of the expression given below

_ _ _      

X.Y.Z + X.Y.Z + X.( Y + Z )

5. In a computer system the ROM :

(a) contains boot software

(b) is permanent

(c) Both of the above

(d) None of the above

Ans. (c)

6. The binary equivalent of 3B7F is

Ans. 0011 1011 0111 1111

7. The register used by the shift reduce passing method is

Ans. Stack

8. A microprogram can be defines as to consist of

Ans. A primitive operation

9.  Find the output for the following C program

    int array[4][4] = {1,2,3,4,5,6,7,8,9,10,11,12,13,14,15,16};

    for (i=2;i<0;i--)

    for (j=2;j<=0;j--)

    printf("%d", arr[i][j]);

10. Find the output for the following C program

#include<stdio.h>

void main()

  {

   int i,x,sum=0;

   int arr[6]=[1,2,3,4,5,6]

   for (i=0;i<4;i++)

   sum+ = func(arr[i]);

   printf("%d", sum);

 }

func(int x)

   {

     int val,x;

     val = 2;

     return(x+ val++);

  }

11. Given the following data:

       Process P1 takes 2 seconds

       Process P2 takes 3 seconds

       Process P3 takes 4 seconds

       Process P4 takes 1 second

Page 69: 7116273 Aptitude IGATE Infosys Tcs Ect

       Process P5 takes 6 seconds

Find the average time in case of shortest job first (SJF) scheduling.

12. Given a string STOCK and a stack of size 4. Which of the following strings cannot be

generated using this stack.

  (a)  TSOCK

  (b)  TOSKC

  (c)  STOCK

  (d)  TKOSC

  (e)  None of these

13. Inversion of a matrix will take which of the following time complexities?

(a)  O(n)

(b)  O(n²)

(c)  O(log n)

(d)  O(n³)

(e)  None of these

14. A drum rotates at 4000 rpm. What is its average access time.

15.  What range of integral values can be stored using 32 bits?

16. Where are the following variables stored

   Automatic

   Global

    Static

17. If a layer 4 transfers data at the rate of 3000 bytes/sec. What will be the size of data block

transferred by Layer 2

18. What is the greatest disadvantage of dynamic RAM over static RAM

Ans. High Power and need to refresh every 2 ms.

19. What happens when the CPU gets interrupted?

20. Find the Postfix of the following string

(a + b) * ((-d) *f (ab - cd))

21. E --> E + E| E * E | E/E | E - E| .... then which is correct

  (a) It is ambigous

  (b) It is inherently ambigous

  (c) It is non inherently ambigous

  (d) None of the above

22. If there are n nodes and K edges in a graph then what is the order of traversing

  Ans. O(n²)

23. A graph is represented as an adjacency list with n vertices and e edges

What is its time complexity

Ans. O(n + e)

24. An array with address KV[a] had n elements. Which of the following correctly addresses the

ith element of the array.

(a) KV(a) - 2a + 2i

(b) KV(a) +2i

(c) KV(a) - 2a

(d) None of these

25. Give an example of a primitive instruction in microprocessors.

26. A computer has 8 bit data bus and 16 bit address line. How many machine cycles will it take

to store the contents to a memory location?

Page 70: 7116273 Aptitude IGATE Infosys Tcs Ect

27. Where is a variable defined in a function stores?

  Ans. Process Swappable Area

28. For the following C progralm

  int d=0;

  for(int i=0;i<31;i++)

  for(int j=0;j<31;j++)

  for(int k=0;k<31;k++)

    if (((i+j+k) % 3)==0)

      d=d+1;

 Find value of d

29.  e=<e+e> | <e*e> | <(e)> | <id>

What forms do the expressions created by the above definition fit in

Ans. All arithematic expressions

30.  If a set of numbers are in sorted order then which of the following sorting method is best

  Ans. Bubble Sort

31. A magnetic tape is similar to which of the following structures

  Ans. List

32. The s/n id 3 dB Find the capacity of the line.

Globaledge

Sample Test Paper  Globaledge

1)what is big-endian.

a) MSB at lower address LSB at higher address

b) LSB at lower address MSB at higher address

c) memory mgmt technique

d) none of the above

ans:a

2)what is Little-endian.

a) MSB at lower address LSB at higher address

b) LSB at lower address MSB at higher address

c) memory mgmt technique

d) none of the above

ans:b

3)8086 has

a)16 bit data bus ,16 bit address bus

b)16 bit data bus,32 bit address bus

c)8 bit data bus,16 bit address bus

d)8 bit data bus,8 bit address bus

ans:a

4) what is the scheduling algorithm used in general operating systems.

a) FCFS algorithm

b) Highest Priority First algorithm

c) Round-Robin algorithm

d) None of the above

ans:c

5)Router is present at

Page 71: 7116273 Aptitude IGATE Infosys Tcs Ect

a)Physical layer

b)Data Link Layer

c)Network Layer

d)None of above

ans:c

6)Condition for deadlock occurance

a) Mutual Exclusion

b) no premption

c) hold and wait

d) circular wait

e) all of the above

ans:d

7)PCI stands for

a)Programmable computer Interface

b)Peripheral Computer Interface

c)programmable Control Interface

d)Peripheral Component Interface

ans:d

8)Toggle state in J-K flip-flop is

a)0 1

b)1 0

c)1 1

d)0 0

ans :c

9)Interrupt is serviced

a)Immediatly when it occurs

b)After the completion of current instruction.

c)Ignored

d)None of the above.

ans:b 

C:

1.what is the o/p ?

void main()

char *mess[]={"Have","a","nice","day","Bye");

printf("%d \t %d",sizeof(mess),sizeof(mess[1]));

a. 16 4

b. 5 4

c. 20 4

d. Error

answer: c

2.what is the o/p of the following programe?

Page 72: 7116273 Aptitude IGATE Infosys Tcs Ect

void main()

{

int i,count=0;

char *p1="abcdefghij";

char *p2="alcmenfoip";

for(i=0;i<=strlen(p1);i++) { 

if(*p1++ == *p2++)

count+=5;

else

count-=3;

}

printf("count=%d\n",count);

a. 15

b. 6

c. 12

d. compiler error

answer: b ( note : strlen(p1) == i when i=6)

3.what does main return on successful execution?

a. 1

b. 0

c. -1

d.Nonzero

answer:b

4.

main(int argc,char *argv[])

{

printf((argc > 1 ? "%c" : "%c",*++argv);

}

If the i/p string is "GESL Bangalore".

a. G

b. E

c. B

d. GESL

answer: c

5. How do u declare a pointer to an array of pointers to int?

a. int *a[5];

b. int **a[5];

c. int *(*a)[5];

d. u con not declare

answer: c

Questions on OS:

1.UDP is a

a. Reliable protocol

b. Unreliable protocol

Page 73: 7116273 Aptitude IGATE Infosys Tcs Ect

c. Connectionless protocol

d. Both b & c

Answer: d

2.Real Time OS uses

a. RISC processor

b. CISC processor

c. SISC processor

d. Any of the above

Answer: a

3.Race condition could be overcome using

a. A global variable

b. A local variable

c. Mutex

d. All of the above

4.Repeaters are in

a. Data link layer

b. Physical layer

c. Network layer

d. Transport layer

Answer: b

5.Telecom Networking uses

a. Packet switching

b. Circuit switching

c. Message switching

d. Non of the above

Ans : b ( normally )

( packet switching in ISDN )

GSSLSample Test Paper

SECTION 1- APTITUDE SECTIONDirections for questions 1-3: Complete the sequence given below1. 5, 5, 13, 13, 21, 21, __Ans: 29

2. 0, 7, 26, 63, 124, __

Ans: 215 ( hint: n3-1 )

3. 1, 3, 5, 7, __Ans: 9

4. If a person walks at 4/5th of his usual speed he reaches 40min late. If he walks at his usual speed for how much time does he travel ?

5. Two trains A&B start at opposite points 120km apart at 60kmph. A fly starting along with train A at 120kmph reaches B then returns back to touch A and continues the two and fro movement. By the time two trains meet how much distance would the fly have travelled?

6. In a class 80% have passed in english, 70% passed Hindi, 10% didn't pass either. If 144 students passed both the subjects. What is the total strength of the class?

7. Find the least number which when divided by 7 gives the reminder 6, when divided by 6 gives reminder 5, when divided by 5 gives reminder 4 and so on ?

Page 74: 7116273 Aptitude IGATE Infosys Tcs Ect

8. If a man stands in front of sun what is the first letter of the direction which is left to him ?

9. (a)A square is to circle as cube is to(b)Success is to failure as joy is to

10. (a)Give the synonyms of the following words(i) Joy(ii) Inert(iii) Jolly

(b) Give the opposites of the following words(i) genuine(ii) command(iii) essential

11.Find the odd man out in the following sets(i) Tiger, Elephant, King Cobra, Dolphin(ii) Oasis, Lake, Pool, Valcano(iii) Bengali, Karnataka, Mumbai, Kashmir(iv) Lapidary, Lancet, Scapel, Surgeon(v) Requiem, Dirge, Elegy, Paean

12. I bought a cycle 2days before my birthday and I broke it 3 days after my birthday. The day I broke it is Mar2,1956?

Directions: The following questions are to be answered on the basis of the above given statementi) When is my birthday?Hint: Keep in mind that 1956 was a leap year.(ii) What is my age on Mar 4th, 1980?(iii) My nephew is born exactly 20years after me. If I turned 20 in 1960, what is the nephews age on Feb 28th 1988 ?

13. Monday Aug25, 96 :Hostess: "Mr A, you forgot your umbrella during the party on last friday. I expected you to collect it on your visit on wednesday as I plan to leave on this Friday."Directions: The following questions are to be answered on the basis of the above given statement(i) when A missed umbrella?(ii)When A is supposed to collect it?(iii)When K leaves?

14.What is my father's sons son to my son?

Ans. Cousin brother

15. On cutting a solid parabola what would be generated

Ans: Cone

16. What is Eulers formula?

Ans: F+V-E =2; whereF => faces; V => vertices; E => number of edges

17. What is Newton Raphson method used for?

Ans: To find the root of f(x) = 0;

18. How many tangents can be drawn on three circles if they don't lie within each other ?

19. xy - x + 2y = 6 equation is shifted to form equation xy=c what is c ?

20.When x is real what is the least value of (x2-6x+5)/(x2+2x+1)

21.When an object like cube or sphere is seen along x, y, z axis we get the same.Apart from these suggest another object which has similar characteristics as that mentioned above?Ans: Triangular prism

22. When an object is seen from the front side we can see two concentric squares and top view also without any hidden lines. Draw the side view.

23. In common parlance, A=> B means whatAns: if A is true B has to be true

23. If A is not invertible and BA = I is not possible, what is implied by this?Ans: Determinant is Zero.

24. What is a free body diagram used for

Page 75: 7116273 Aptitude IGATE Infosys Tcs Ect

25. A die is thrown twice what is the probability that you get same number

26. The sum of two numbers is 55. What is the larger number?

SECTION 2-TECHNICAL SECTION

1. Convert 251 in base 10 to octal(base 8)?

2. How much information can be stored in 1 byte of a IBM pc compatible?

3.What is the language used for Artificial IntelligenceAns: lisp

4. Swap two variables without using temporary variableAns: a= a+b; b= a-b; a= a-b:

5.Which is not the operating system ?Ans: BIOS

6. What is the optimum number of operations for 2x3 + 3x2 + 5x + 5?

7. In the fortran language which of the following is true.(i) fortran uses call by value(ii) fortran is object oriented(iii) fortran allows use of function overloading

Ans. (i)

8. When a program is compiled what does it produce?Ans: Source code is converted to object code

9. What is the difference between function overloading and function overriding?

10. What is the character set used in JAVA 2.0 ?Ans: Unicode

SECTION 3 - C TEST1. What is the mistake in the following program segment ?

f(){int a;void c;f2(&c,&a);}2. a=0;   b=(a=0)?2:3;

a) What will be the value of b and why ?b) If in first statement a= 0 is replaced by a= -1, b = ?c) If in second statement a=0 is replaced by a = -1, b=?

3. char *a[2];int const *p;int *const p;struct new { int a;int b; *var[5] (struct new)}

Describe the statements in the above given construct ?

4. f(){int a=2;f1(a++);}f1(int c){printf("%d", c);}What is the value of c ?

5. f1(){f(3);}

Page 76: 7116273 Aptitude IGATE Infosys Tcs Ect

f(int t){switch(t);{case 2: c=3;case 3: c=4;case 4: c=5;case 5: c=6;default: c=0;}What is the value of c?

6. What is the fallacy in the following program segment ?int *f1(){int a=5;return &a;}f()int *b=f1()int c=*b;}

7. Give the C language equivalents of the followinga)Function returning an int pointerb)Function pointer returning an int pointerc)Function pointer returning an array of integersd)Array of function pointer returning an array of integers8. Find the fallacy in the following program segment?int a;short b;b=a;

9. Define function ? Explain arguments in functions ?

10. How does C pass variables to a function ?

11. Explain the following program segment.f(){int *b;*b=2;}

12. Explain binary trees and their use ?

13. Draw the diagram showing the function stack, illustrating the variables that were pushed on the stack at the point when function f2 has been introduced .type def struct{ double x,double y} point; }main( int argc, char *arg[3]){ double a;int b,c;f1(a,b); }f1(double x, int y){point p;stack int n;f2(p,x,y)}f2(point p, double angle){ int i,j,k,int max;}

Hellosoft

Sample Question Paper

1. What is diners phylosophers algorithm. 2. What is atomicity.Explain 3. Give an example of source- level debugger in unix/linx. 4. What is Pre-emptive multitasking. 5. Do not remember this one.

DataStructures: (ONly 5 Questions)

Page 77: 7116273 Aptitude IGATE Infosys Tcs Ect

1. Where will the parameters be stored when a function is called in a program. 2. What is recursion?What are its disadvantages. 3. Which one is the best and efficient sort?

Networking:(Only 5 Questions)

1. ASN.1 belongs to which layer. 2. Give an example of popular Transport Layer Protocol 3. What is the netmask of Class B IP address? 4.  Explain CSMA/CD. 5. Given a size from 0-7 what would be the least window size.

C Language (Total 13 Questions)

1. main() {  printf("%d",printf("HelloSoft"));} Output?

2. case 1: case 2:      typedef Struct { typedef Struct {                 int a; char p;                 char b; int q;                 int d; char k;                 char e; int l;                  }A; }A;     Assuming 'packing' is not enabled, which case will give an error of Sizeof(A) less.

3.  main()  {   int i=3;   printf("%d %d %d",i++,i,++i);    }

4. main()     {       int i=10;      int j,k=5;      int a[10];      for(j=0;j<10;j++)      a[j]=(i+k)+(i*k);    } Optimize the above code.

5. Write In Order Tree Traversal Pseudo-code. 6.  main()

    {   int *p=0x100;   int *q=0x100;   int k=p*q;   printf("%x\n",k);   } Output ?

7. Char* foo(Str...) {     char str[4];     strcpy(str,"HelloSoft");     return str;      } Output?

8. int a[10][20][30][40];       int *p        How to access an element of a using p?

9. main()   {      int i=10;      if(i>20)      if(i==10)         print("Hi");       else          printf("Bye");     } Output ?

10. main()   {    float f;    int i;    //something like this   not remember these 4 questionsexactly    f=(float *)malloc(sizeof((float *)*4));   }         Some Question was asked  not remenber .

HFCL

Page 78: 7116273 Aptitude IGATE Infosys Tcs Ect

HFCL CAMPUS TEST [1] 

(1) TTL to CMOS pullup & vice-versa

(2) function of Ping

(3) SNMP protocol

(4) Application layer

(5) ## ----> token pasting in C

(6) Question on C: i^=j you have to tell the output given the values of i and j;

(7) Complexity of Binary Search tree

(8) Macro is given as follows : SQR(x) x*xfind the value of SQR(i-j) , when i and j were given

(9) Multiplexer 1 to 3 inputs--- A,B,C control lines--- S1,S2 Output --- Y .Write Y interms of S1,S2

(10) Bandwidth minimum sampling theorem

(11) Shannon's Law

(12) Difference between fork() & execve()

(13) Banker's algorithm

(14) append(&struct , fp)

(15) Windows95 , X-Windows what are they?

(16) 16k memory , ending address of the address space FFFFH find the starting address?

(17) Difference between TCP/IP and OSI model

(18) Difference between Bridge and router

(19) A graph was given : " a qudrilateral with 1 diagonal " find the number of trees in the graph

(20) Difference between TCP and UDP

(21-23) Pass by reference in C (3 questions)

(24) Synchronisation technique for IPC ( probably which method among the 4 choices was best was asked)

(25) Address of RST 6.5

(26) Preorder Traversal

(27) Infix to Postfix

(28) What data structure is used to transform infix to postfix?

(29) 4 bit computer , size of ROM given , output of ROM gives the address , design a mod-n counter

(30-31) FIFO , LRU page trace

(32) How many flip-flops in MOD-33 counter?

(33) printf("\nab\bcd\ref");

(34) doorom hello test find argc and argv

(35) call main()main(){toupper("Hello");main();}

what does this piece of code does?

(36-39) 4 question from Understanding Pointers in C on Pointer , Array , all from exercise in the book

(40-41) 2 questions on precedence rule

Page 79: 7116273 Aptitude IGATE Infosys Tcs Ect

HFCL CAMPUS TEST [2] 

(1) ICMP protocol which layer?

(2) Ping - which protocol implemented?

(3) FTP - which protocol implemented?

(4) What kind of handshaking is used in TCP/IP?

(5) IP addressing ClassA , ClassB , ClassC

(6) End to end error correction is done by which layer ?

(7) Question on address bus and data bus ALE 

(8) 16 bit address bus and 8 bit data bus , how much memory can be interfaced ?

(9) 1 bit opcode , (n-1) operand , find the length of Program Counter

(10) 4 bit ROM

(11) What is the benefit of memory interleaving ?

(12) How many maskable and non-maskable interrupt in 8085

(13) RISC what kind of Control Unit is implemented ?

(14) In Pipeline architechture , what is the value of CPI (Cycles Per Instruction)

(15) Where TLB is used?

(16)

struct a{char b[7];char *s;};struct b{char *t;struct a y;};struct b q={"Raipur" , "Kanpur" , "Jaipur"};printf("%s %s" , q.t , q.y.s);printf("%s %s" ,++q.t , ++q.y.s);Find the ouput

(17) Heapsort worst case complexity

(18) Binary search complexity

(19) Where far pointer is used?

(20) Why address of operator(&) cannot be used on register variables?

Induslogic

Induslogic Test  Sample Question

1. What is the output of the following code when compiled and run? Select two correct answers.

public class Question01 {

    public static void main(String[] args){

    int y=0; //line 1

    int x=z=1; //line 2

    System.out.println(y+","+x+","+z); //line 3

      }

}

A. Prints 0,1,1 

Page 80: 7116273 Aptitude IGATE Infosys Tcs Ect

B. Error during compilation at line 1 

C. Prints 0,0,1 

D. Error during compilation at line 3 

E. Error during compilation at line 2

2. Question 2. Select three correct statements.

A. The garbage collection thread cannot outlive the last user thread. 

B. The garbage collection can be forced by invoking System.gc(). 

C. The garbage collection thread is a non-deamon thread. 

D. The finalize() method is invoked at most once by the JVM for any given object. 

E. The finalize() method may resurrect the object upon which it has been invoked.

3. What is the output of the following code when compiled and run? Select one correct answer.

import java.io.*;

  public class Question05 {

  public static void main(String[] args) {

 Question05Sub myref = new Question05Sub();

  try{

  myref.test();

   }catch(IOException ioe){}

 }

  void test() throws IOException{

  System.out.println("In Question05");

  throw new IOException();

   }

}

  class Question05Sub extends Question05 {

  void test()throws IOException {

  System.out.println("In Question05Sub");

}

}

A. Prints: 

     In Question05Sub

B. Prints: 

     In Question05

C. Prints: 

     In Question05

     In Question05Sub

D. Prints: 

     In Question05Sub

     In Question05

E. The code does not compile.

4. Select two correct statements about the code given below?

class A{}

class B extends A implements E{} //line 1

class C extends A{}

class D extends B{}

interface E{}

public class Question07 {

public static void main(String[] args) {

A a = new D(); //line 2

C c = new C(); //line 3

E e = (E)a; //line 4

B b = (B)e; //line 5

Page 81: 7116273 Aptitude IGATE Infosys Tcs Ect

}

}

  A. The code compiles without error and runs fine. 

  B. Compilation error on line 1 because interface E is not yet declared (forward-referencing). 

  C. Compilation error on line 4 because class A does not implement interface E. 

  D. The cast on line 4 is mandatory. 

  E. The cast on line 5 is not mandatory.

5. How many objects are eligible for garbage collection immediately after line 1? Select one

correct answer.

public class Question08 {

public static void main(String[] args) {

Question08 q08 = new Question08();

q08.doSomething(); //line 1

Thread.sleep(20000);

}

public void doSomething(){

Object[] objArray = new Object[2];

for(int i=0;i<objArray.length;i++){

objArray[i] = new Object();

}

}

}

A. 0 

B. 1 

C. 2 

D. 3 

E. 4

6. What is the output of the following code when compiled and run? Select one correct answer.

public class Question09 {

public static void main(String[] args) {

try {

int i = (int)(Math.random()*10);

if(i<=5)

System.out.println("i = "+i);

else

throw new Exception("i > 5");

} catch (Exception e){

System.err.println(e.getMessage()+" (i="+i+")");

}

}

}

  A. The output cannot be determined. 

  B. Compilation error. 

  C. An exception is thrown at runtime. 

  D. Output is i = 2 

  E. Output is i > 5 (i=6)

7. What is the output of the following code when compiled and run? Select one correct answer.

public class Question10 {

public static void main(String[] args) {

new Question10().doSomething();

}

public void doSomething(){

Page 82: 7116273 Aptitude IGATE Infosys Tcs Ect

int i=5;

Thread t = new Thread(new Runnable(){

public void run(){

for(int j=0;j<=i;j++){

System.out.print(" "+j);

}

}

});

t.start();

}

}

A. Prints 0 1 2 3 4 

B. Compilation error. 

C. No output. 

D. IllegalThreadStateException is thrown at runtime. 

E. Prints 0 1 2 3 4 5

8. What is the output of the following code when compiled and run? Select one correct answer.

public class Question11 {

public static void main(String[] args) {

StringBuffer buf1 = new StringBuffer("Hello W");

addSomeStuff(buf1);

System.out.println(buf1.toString());

}

public static void addSomeStuff(StringBuffer buf){

StringBuffer b = buf.replace(6,10,"orld");

System.out.println(b.delete(0,1).toString());

}

}

A. Prints 

     Hello World

     Hello World

B.  Prints 

     Hello orld

     Hello orld

C. Prints 

    Hello orld

    ello orld

D. Prints 

    ello orld

    ello orld

E.  Compilation error.

9. What is the output of the following code when compiled and run? Select two correct answers.

(Note: when an instance of a Vector is printed, its content appear between square brackets [])

import java.util.*;

public class Question13 {

public static void main(String[] args) {

Vector col = new Vector();

col.add(new Integer(1));

col.add(new Integer("2"));

col.add(new Float(3.2d)); //line 1

col.add(col.elementAt(1));

col.setElementAt(col.elementAt(2),0);

Page 83: 7116273 Aptitude IGATE Infosys Tcs Ect

System.out.println(col);

}

}

A. Compilation error on line 1. 

B. Only line 1 won't compile. 

C. The code compiles and runs fine. 

D. Prints [3.2, 2, 3.2, 2] 

E. Prints [1, 2, 3.2, 2]

10. Select three correct statements.

A. A static method may override another static method. 

B. A static method cannot override a non-static method. 

C. A non-static method cannot override a static method. 

D. A non-static method may be overloaded by a static method. 

E. A synchronized method cannot be overridden.

IntegraInetrgraph Software Analytical 

1. Given a cube, with different colors on its faces, and then is cut into 64 pieces, and the questions relate to the colors of different  colored small cubes.

2. A few ladies and gents sit around table in some given order and 4 questions are about their seating arrangement with some restrictions.

3. Draw a venn diagram for 3 items : white, flowers, clothAns : draw 3 circles each intersecting the other , with white kept in the middle.

4. A problem related to seating arrangement of 4 people ( 2 ladies and2 gents) with some restrictions

5. problem related to milk with water added to it for three timesAns: 20.5 litres

6. Problem related to diagrams . Five diagrams were given and asked tofind the sixth one.

C-Skills 

1. Max number of nodes in a binary tree with height 3 is 20 : Ans: False

2. 10,20,30,40,50,60 : give the order when put in a queue and in a stackAns : Queue : 10,20,30,40,50,60stack : 60,50,40,30,20,10

3. Debugging is the process of findingAns : logical and runtime errors

4. trace the error:void main(){int &a;/* some other stuff here */}Ans: syntax error

5. a problem with a function named 'myValue' will be given and asked to find the value of main() for an argument of 150,Ans : 150

6. Some problems related to 'for' loops.

7. problem on conditional inclusion.

8. problem related to depth first and breadth first search (DSA subject)

9. study the syntax of malloc function

10. locate the errorint arr (20);Ans: syntax error. 

Page 84: 7116273 Aptitude IGATE Infosys Tcs Ect

Intera Software

Interview Procedure

The information on the interview is pretty sketchy but it may consist of both technical grilling and HR interview.Written Test Paper consists of two sections : 1. analytical (20 marks)2. C skills (20 marks) total time 45 min

InterwovenInterWoven:

This company is in Bangalore and they conducted a telephonic interview to screen the people at first level. This is the Technical interview conducted for 3+ years experienced in Java, J2EE.__________________________________________________

1. Why do you want to leave current company ?

2. Explain MVC architecture and functionalities of various components ?

3. I have a file of very very large file size at client side, and I have a JSP page. Using this JSP page, if I want to send the file to a servlet (this servlet will store it somewhere), what is the best method to do it ?

4. What is the difference between normal beans and EJBs ?

5. How system level services in EJBs are managed ? And tell about Deployment Descriptor ?

6. What are various types of EJBs ?

Sasken

SASKEN COMMUNICATION INDIA

There are 10 ques. in three sections each, thus total 30 ques. 4 choices were given. 1 mark for each ans. no negative marking.

1 if phase of different frequency components of signal vary linearly then they all reach destination with delay1) delay also vary linerly.2) no change in delay,3)

2 what is the relation in dbm and db?1) same2) dbm is 10 times db3) db is 10 times dbm4) db is relative measure and dbm is absolute measure.

3 difference in PSK and QPSK a. slower sample rateb. greater sample ratec. slower bit rated. greater bit rate.

4 what is the main problem with power line communicationa. high audio signal generationb. seperation of modulating and career signalc. (something) low pass filtering

5 a signal is to be transmitted 1000km long distance sending 10 mbps . if signal consists of 1000 bits/package and takes 5 microsecs. to transmit then how much data is to be send such that line gets fully filled.a. 100b. 500cd.1000

6 if a process is wide sense stationary then a. it's probability density function is also wssb. power spectral density is wss

Page 85: 7116273 Aptitude IGATE Infosys Tcs Ect

c. it's mean constant and auto correlation function depends only on time diff. t1-t2.d.

7 for intelligent sound signal transmission a. low pass filter of 1.8khz is to be used.b. hpf of 1.8khzc. d.

q.8 if a 32 bit number in fractional form is to be converted into float then it is multiplied witha. 2^31b. 1/(2^31 - 1)c. 2^31 -1d.

9 if signal is sampled at 8 khz and the signal of 1 khz is transmitted then

q.1 char const *p='p' then which of below is truea.**P = (this question u can get from freshersworld sample paper)

q.2 recursive function count(n){if count<=1then result = 1;else result = count(n-3) + count(n-1) }for count(6) ans is:a. 3b. 6 c. 9d. 12

q.3 for(i=0;i<=50;i++)for(j=0;j<=100;j++)

for(j=0;j<=100;j++)for(i=0;i<=50;i++) for this type of loop which one is fastera. sameb. firstc. second d. depends on s/w and h/w

4 #define putchar(%f,c)int c='d';putchar(c);what is the output?a. ,b. some numbersc. compiler error d. segmentation error

5 file atest(){ int a; }

file bvoid main(void){ int b;b= test();}what is the outputa. ab. bc. not compiled,errord. compiles sucessfully,but give linker error

q.1 if 2<X<ya. (x^2)*yb. y^2)xc. 5xy

Page 86: 7116273 Aptitude IGATE Infosys Tcs Ect

d.e.

2 in parallelogram what values of angles r possiblei. 90,90,90,90ii. 70,130,70,130iii. 40,70,150,130 a. i and ii onlyb,c,d,e : combinations

q.3 if x+y=z is given what can be inferred from it0i. x<Zii. x<Yiii. x<2za. i onlyb. i and ii only n ohter combi

4 if B is 8 miles in east from city A, c at 16 in north from B, D 16 miles in east of C, E at 12 in north of D then what is the distance b/t a and e.a. 10 b. 20c. 30 d. 40e. 50

5 in a library for each single novell there r 2 science book and for each science book there r 7 economics books then the ratio of economics to science to novell isa. 1:2:14b. 2:1:7c. 14:2:1d.e.

q.6 ira is taller than sam, harold is shorter than gene, elliot is taller than haroldsam and harold is of same height. thenwhich one is true.a.sam is shorter than harold b. elliot is taller than genec. ira is taller than elliotd. elliot is shorter than gene e. harold is shorter than ira

7 if a(n) is to be entered following the rule [ a(n)^2 - 1 ] then what r the next three enteries.a. 0,-1,0 b. 2,3,4c. 1,2,3d. 0,-1,2e. 0,-1,1The test comprises of 2 sections: 

1. Technical ( C ) & Technical Subject- 60 mins, 60 questions  2. Logical Reasoning.. - 30 mins,17 questions....

IKOSIndia

IKOSThere were three sections:1. Software2. Hardware3. Analytical  Only one MCQ question; all others to be solved analytically.  1.) Each of the letters below represents a non-zero and unique digit.Findthe digit.     one     one     one    +one    -----      tenFind the digits.  Ans( one = 182 , ten=728).

Page 87: 7116273 Aptitude IGATE Infosys Tcs Ect

2. Calculate the maximum permissible propagation delay of each FF in amodulo 1023 counter.3. Find the error if any -  {struct *sPtr, *nPtrfor(;*nPtr;nPtr=nPtr->next)    free(nPtr);}

4. Find the output -  main(){ void f(int, int);   int i, j;   i=10;   j=10;   f(++i,j++);}

void f(int i, int j){if(i>50)return;  else   i+=j;  f(i,j);  printf("%d",i);  }

Ans 413121 5. Implement the following usin 4X1 MUX.   A'BC + AB'C +ABC'     6. How will you convert an S-R FF into J-K FF ?

PHILIPS

PHILIPS PAPERQ1)for all X(roar(X)=3D>lion(X))   a)all lions roar   b)some lions roar   c)   d)Q2)on some polynomial boundsQ3)on emiter coupled logicq4)four qustions on  given digital ckt  from kennedy book ther are 5 questions.

Q5)one from  recivers,one from digital comm,one coaxial cable repeaterdistace depends on the channel BW.What is the use of IFQ6)If channel BW is 35khz.What is the maximum freq of data u cantransfer.=3D70khzQ7)line with 2400buads,the dat u can ransmit on it isa)1200kpbsb)2400kpbsc)96kpbsd)192kpbsthese are the qustions in 1 st section         section2----------------------------------------------------------------------Q1) void(int *a,int *b)  {    *b=3D*b^*a;  /* ^ is exclusive OR */    *a=3D*b^*a;    *b=3D*b^*a;    *a=3D*b^*a;    *a=3D*b^*a;  }  this function gives the value  a)a & b values swaped  b) a&b  unchanged  c)  d)Q2) on inorder traversal in binary treeQ3) on black box testingQ4) on fun(n)   {      unsigned long n=3D~0; /* ~ is ones complement */

Page 88: 7116273 Aptitude IGATE Infosys Tcs Ect

       for(i=3D0;(n>>1)!=3D0;i++)    }  Out put of this programme segment is  a)it will give the word length in that machine  b)gives max int vlue in the machine  c)  d)Q5)                           section3---------------------------------------------------------------------Q1)sentenc given with blanks we ahve to fill tehm with wordsQ2)same as aboveQ3)A qustion on relations   david is grand father to sue   karen is sister to jim   jim is uncle to eric and sue   jim is nephew to larryonly married couple can have children and blood relation can't marrythere are 4 questions on thisQ4)on data suficiencythey will give the table of data    philips  bpl  onida  vediocon---------------------------------1989   data    "   "        "'""'

  q1)whose is consistent growth  q2)whose is highest groth  q3)lowest growthQ5)GK question  ambassidor meansQ6)who IS WHERE  ANS: hagueQ7)ALIAS:NAME::-----------Q8)JOKE:LAUGH::-----------Q9)they will give u diagram and find the shortest path between somepoints.like belman ford algo in NW.

 

Section 1 - Electronics and Mathematics - 20 questionssection 2 - Computer Sciencesection 3 - Aptitude

Electronics and computer science

1. A circuit was given with lot of flip-flops etc and the operation of thatcircuit was asked.    2. 15 software functions are there.  It is known that atleast 5 of them aredefective.  What is the probability that if three functions are chosen andtested, no errors are uncovered.     Ans : 10  *  9  * 8       --    --    --                 15    14    13            

Computer Science 1.Java is         a) Multithreaded b) intrepreter c) compiler d) all of the above           Ans :d2.  The number of nodes in a k-level m-ary binary tree is :

AptitudeA graph was given and questions regarding shortest path was asked.For one question shortest path was asked and the answer is none of theabove.Overall the question paper was easy.

Interview: 

HR questions:      1.What is your strengths and weaknesses2.What are the values u respect  

Page 89: 7116273 Aptitude IGATE Infosys Tcs Ect

3.Site a reason why philips should hire u4.What will u do if u are asked to manage a project which will definitelyskip its deadline.

Technical (for me)1.What is runtime locatable code?2.What is volatile, register definition in C3.What is compiler and what its output.

TELCO

TELCO PAPER - JAN 20061) Hypothesis :problem (below 4 he will give)2) Mirror:Image3)money:Misapprobation4)nackles:Adoration5)construction:building6)file:pile7)ours:we8)1/3,1 1/3,3,5 1/3, next9)Selling price of 4 articles=Cost price of 3 articles then %loss(ans. 25%)10)|x-3|=3-x then x=11)data sufficiency p>q?1)p,q positive2)q-1=q*2+pThis type you can see in GMAT book.Reasoning1) ABCDEF attended for an interview, in which 3 were selectedA is worst of the lotC got equal marks as F(C=F)DE is not selected B>C<!--[if !supportLineBreakNewLine]--><!--[endif]-->

There are 5 questions below this.2)There are 7 fellows sat in the following way C and F always sit as a apart as follows No. of fellows sat b/w c and D and D and F are equal like that he has givenAns: Order is as follows C E B D A G F by using this you can answer all About 4 to 5 are there<!--[if !supportLineBreakNewLine]--><!--[endif]-->

3)ABCDE are five brothers. There are twins(only one pair of equal age) both are neither younger nor older. D is younger to 3 brothers.B is older than E and C. q's like who is youngest? Eldest ? About 4 are there.<!--[if !supportLineBreakNewLine]--><!--[endif]-->

4)A person has meet a king for that ha has to cross 7 gates. At each gate he has to pay half the amount he is carrying. Finally he gave Rs 3/- to the king. Then the amount he carried at the beginning and some questions (about 5)like that.<!--[if !supportLineBreakNewLine]--><!--[endif]-->

5) Heros tell truth and cowards lie.There PQR three persons. P tells Q "I may be hero or I may be coward". Q tells R "P was telling that he was coward. Then R tells Q "P was not a coward but he was a hero". you better study the question it may not be entirely correct.In thus 3 q"s are there

Part 1: (Verbal/English, 25 Q - 15 min)This was cool.All Antonyms, Analogies got repeated from previous papers.

Antonyms:1)equanimity2)sequester3)apathetic4)dislodge5)sedate

Analogies:1)celeberate:marriage::a. window:bedroom b. lument:barevemantc. pot:pan

Page 90: 7116273 Aptitude IGATE Infosys Tcs Ect

d. face:penalty2)neglegent:requirement::remises:duty cognet:argumenteasy:hard careful:position3)Germ:disease::man:women doctor:medicinewar:destrustion shopkeeper:goods4)bouquet:flower::skin:body chain:linkproduct:factory page:book5)letter:word::club:people page:bookproduct:factory picture:paper

Time is sufficient.No special preparation needed (provided u r good enough)Just gaze thru previous papers' antonyms & analogiesSentence completion & fill ups are easy.Instead of mugging up antonyms, I personally recommend u to read Synonyms, so that u can easily locate their opposites. (U can refer TCS papers also if needed)

********************************************Part 2: (Quantitative Aptitude, 45 Q - 30 min)

This was tough.

Repeated Q's:

One monkey climbs a poll at the rate of 6mts/min and fell down 3mts in the alternately. Length of the poll is 60 mts , how much time it will take to reach the top?a. 31 b.33 c.37 d.40 (ans: 37)

X men work for X days to produce X products, then Y men can produce Y products in - - - - days. (ans: y^3/x^2)

successive discounts problem

sqrt(12 + sqrt(12) + ((sqrt(12) +......................infinity) = ? (ans: 4)

6 locks & 6 keys (permutation) problem

Other Q's:

consider a square ABCD, in which E is the mid-point of BC & F is that of CD. Now find the ratio of area of triangle AEF to the area of square ABCD. (ans: 3/8)

Do not remember rest of the them.

Bottom Line:This section makes the difference.Special preparation is mandatory.Try to solve problems of every type from R.S. AgarwalLeave complex Q's & proceed forward.

********************************************Part 3: (Reasoning, 30 Q - 30 min)

This was 50 - 50I confidently solved around 16 of them.

1)There are 4 buses - A,B,C,D. There are 220 students in a school.A can carry 60 students. B can carry 50 students. C can carry 40 students. D can carry 35 studentsCost of travelling in the 4 buses were given, A - 160 , B- 140, C- 125 , D- 95 (not exact values)a) Find the bus combinations, so that all the students can be carried in the minimum cost (One can use any no. of buses of a particular type)b) Find the min. no. of buses required to carry all studentsetc.

2) ( not exact )P speakes Italian & FrenchQ speakes Spanish & English

Page 91: 7116273 Aptitude IGATE Infosys Tcs Ect

R speakes Italian & GermanS speakes Spanish & FrenchT speakes English & German etc.

Find a) Mediator between P & Qb) Most popular languageetc

3) All P's are Q'ssome R's are not C'sSome C's are P's& so on ( 5 Q's based on these facts)

4) One Logical Venn diagram problem

5) One simple flow chart

Go thru R.S. Agarwal(Verbal Reasoning) & then Barrons(Analytical Reasoning Only)

(Paper Contributed By: Viswanadh Kattepalli)

Ivega

IVEGA PATTERN - OCT 2003 - BANGALOREHi ,

After a time of 2 and a half months of completing graduation iam finally employed as a software engineer at Ivega.(Bangalore) I shall be joining on the 27th of this month.

I thank  all the chetana group members including chetana and suri sir for the contribution they made by informing abt various openings at various places while i was job hunting.

Ivega selection process is as follows:1st  Round: Written test (English,arithematic reasoning,General knowledge)2 nd  Round: Group Discussion (Is education correlated to success i.e. is education required to achieve success in life or not)3 rd Round: HR interview4th Round : Technical round (Questions on probability if you select maths as your favorite subject)Iam an BE- ECE student and was not asked much technical on c and c++ except basics .5th Round: This is just a round for formality.Questions like 1.What does commitment mean to you.2.Do U have a role model ?Who is it and why.3. Which is better Hardworking or smartworking.4.are you planning for higher studies?They will inform abt the rules and regulations of the company and ask if it is O.K.with U.That's it.

CaterpillerCaterpillar-2004 FEB Sample Paper

Verbal:

1. Synonym: pontificate. (Talk in a dogmatic and pompous manner)

2. Antonym: loquacious. (Means Full of trivial conversation)

3. Analogies: Whiff: scent Birds: anthropology (doubtful) Generator: Current.

vacilliate :resolution as diobey :order…

4. odd man out: iconoclast,hyperbole,exaggeration amused ,bemused, preoccupied mural,nostalgia,dream

General Awareness:

1.Myopia

2.Scurvy

3. Montagu-chelmsford reforms

Page 92: 7116273 Aptitude IGATE Infosys Tcs Ect

4. How can you remove the president from his office?

5. Minimum age for a person to be eligible to become a president.

6. Who can preside over the meetings of both the sabhas but still cannot vote?

7. regional army command is commanded by

8. strength of defenc epersonal is 10 lakhs ,12lk,15,lk

9. mirage 2000 aircraft is from which contry

10. broad gauge 1.67m,2m,1.83m

11. production of steam locomotives stopped from year

12. first presidensy ban in which city,Chennai,Mumbai,Calcutta,delhi

13. export import bank started in yr

14. what % of Re,1 coins in Indian currency

15. what earns more  foreign money,tea,jute…

16. strength of indian army.

17. who commands the regional army.

18. mirage bought from ans:france

19. inflation occurs due to (increase in money)

20. from where india get max for exchange.(jute,iron steel,sugar,tea)

21. % of 1 rupee coin in total indian currency.

Tech:

1.Which of the following uses consumable electrodes.(I believe the options were MIG,TIG and stuff like that)

2.Alloy used in the manufacture of surgical instruments.

3. Uses of superconductors.

4.Four questions from IC engines :

a. For a given compression ratio the efficiency of Otto cycle>Diesel cycle.(Is this right?)

b. Turning moment diagram for compression stroke of IC engine.

5. What path does double slider crank mechanism follows. (Check if it is Elliptical).

6.Ratio between Brake horse power to the energy supplied.

7. Two emfs E1 and E2 are given. Among two lengths one is given. Be clear with the relation between ratio of emfs to the lengths across.

anodizinghoningthermoplasticif fine powder is added to metal soln,it forms single crystal/ refines grainsfunction of draft allwance in mouldingif various sizes ,hp….etcof engines r to be tested the which measure is taken a.a/f ratiob.swept vol etc

value of pressure used in elec resistance weldingup millingdown millingone of the tool angles definition is give \n choosehammer blowpump in sewagepump in oilheat trnfer based oin which thdy lawsum usin thermal coeff of exp to fin stressporytion hot in refridegerationspinning causes bending or bending and stretching or..critical speed of a shaft is affected by

Page 93: 7116273 Aptitude IGATE Infosys Tcs Ect

starting friction is low inht transfer is based on which law(zeroth,first,2nd,kirchoff)in refregeration where will temp be max?ans:between compres and condenserstoichiometric AF ratio is (chemically correct mixture by wt,ans:chemically correct mixture by efficiency)anodidising ishow will u compute performance of desel eng?ans:bsfchow will u pump sewagw water ie which pump?how will u pump viscous fluid?honing process:if thread angle is increased (mechanical adv decreases)principal plane condition:(shear stress 0,normal stress max)if refrigerator is opened in a room ,fan is also on.what will happen to room temp?graft in castinghow will u improve resistance

Quantitative:

1.Two trains are travelling between two stations at a constant speed of 25kmph.The stations are 50km apart .A bird traveling at 100kmph starts its flight along with one train and travels in the same direction until it meets the other train. It changes its direction and goes in the reverse direction until it meets the initial train. Find the distance travelled by the bird before the two trains collide.

2.A flight carrying food packets is flying at a height of 1960m from the ground. The speed of the flight is given. Find the time taken for a food packet dropped at that instance to hit the ground.

(Hint: Use s=ut+(at^2)/2.Initial velocity as 0 coz the dropped food packet does not have velocity in the vertical direction.)

when a square is constructed on 3 sides of a triangle ,their sides are in the ratio 1:3:4,THE WAT triangle is itx2+y2=45,xy=18,then 1/x+1/y=x-11/x>13/15,then x=in race 100m,b bet c by 25m.a bet b by 20m.then a will beat c by what metres5 tables and 10 chairs were sold for 20000.15%gain for table,10%loss on chair.overall profit rs 750.the cp of table isa man rows in still water in 9kmph in 2hr.if upstrm vel is 2times down,then find speed of streamh:base dia of a cone is 3:2.icecream is filled usig hemisph scoop whose half part is inside cone.if total length of icecrem is 36 cm,find vol of icecreama,b,c complete work in 4 days.a &b finifh in 7 days.time for c is & similar sumslength of line perpendi to given line eqnavg of 5 playrs is x.addition of 6th player increases avg by 2.add od 7thplyr decreases avg by 2.runs of 7th plyr isif 500 more cows r there then each cow gets 40% less hay.the no of cows isvalue of gold is increasing by 11%.if present value is 10000,then value beforwe 2 months isvenn diagram verbal suman amount doubles in 5 yrs due to SI.wat will be the value of the amt.in 13yrs 9 months if princi is adden every yearfirst a=Ivaue of a sqre is assigner to bc=b+1if sqrt of c>5o terminate,else goto beginningwat is the last value of c22. sum of sqr of 2 nos is 45,prod is 18 find the sum of the reciprocals.

23. labour sum(6 days)

24. Cows consume hay.500 more cows decreases the hay by 40% per head. find the no of cows.

DELPHIDelphi-TVS 23 APRIL 20 03  ALAHABADTESTS:100 mins 130 qns.

Directions: For questions 1 to 5. Each sentence below has one or two blanks, each blank indicating that something has been omitted. Beneath the sentence are four lettered words or sets of words. Choose the word or set of words for each blank that best fits the meaning of the sentence a whole.

Though science is often imagined as a _______ exploration of external reality, scientists are no different from anyone else: they are ______ human beings enmeshed in a web of personal and social circumstances.a) fervent..vulnerable b) neutral..rational

c) painstaking..careless d) disinterested..passionate

Among the many ______ of the project, expense cannot be numbered; the goals of the project promoters can be achieved with impressive ______.a) highlights..efficiency b) features..savings

c) disadvantages..innovation d) defects..economy

A leading chemist believes that many scientists have difficulty with stereochemistry because much of the

Page 94: 7116273 Aptitude IGATE Infosys Tcs Ect

relevant nomenclature is ______ in that it combines concepts that should be kept ______.a) obscure..interrelated b) impressive..discrete

c) subtle..inviolate d) descriptive..seperate

The old man could not have been accused of ______ his affection; his conduct towards the child betrayed his ______ her.a) lavishing..fondness for b) sparing..tolerance of

c) rationing..antipathy for d) stinting..adoration of

Many of the earliest colonial houses that are still standing have been so modified and enlarged that the ______ design is no longer ______.a) pertinent..relevant b) initial..discriminable

c) embellished..attractive d) appropriate..applicable

Directions: For questions 6 to 10. In each of the following question, a related pair of words or phrases is followed by four lettered pairs of words or phrases. Select the lettered pair that best expresses the relationship similar to that expressed in the original pair.

MISER : THRIFTa) performer : artistry b) chauvinist : patriotism

c) mimic : ridicule d) politician : compromise

VOTING : ROLL CALLa) termination : cloture b) amendment : constitution

c) majority : concession d) quorum : filibuster

ARTICULATE : CLEARLYa) orate : strongly b) shout : loudly

c) lecture : willfully d) malign : incoherently

TALON : EAGLEa) fang : snake b) hoof : horse

c) claw : panther d) quill : porcupine

INSULIN : PANCREASa) bile : liver b) menthol : eucalyptus

c) oxygen : heart d) honey : bee

Directions : For questions from 11 to 15. Each question below consists of a word printed in capital letters, followed by four lettered words or phrases. Choose the lettered word or phrase that is most nearly opposite in meaning to the word in the capital letters.

VAGUEa) expressive b) Felicitous c) well-defined d)nearly perfect

FOCUSa) disappear b) disperse c) link d) activate

PROLOGUEa) soliloquy b) trilogy c) analogue d) epilogue

DISARMa) hold close b) put on guard c) challenge d) entertain

INFLATEa) converge b) inhibit c) audit d)minimize

In a telecommunication – cable assembly plant, cables are assembled by twisting plastic – coated wires together. There are wires of exactly six different solid colours – red, yellow, violet, green, white and black. Wires must be assembled into single cable according the following rules.

Each cable must contain at least three wires and wires of atleast three different colours.

At most two wires in a single cable can be black

At most two wires in a single cable can be white

There can be at most one wire of each of the other colours in a single cable

If one wire is red, then one wire must be yellow.

Page 95: 7116273 Aptitude IGATE Infosys Tcs Ect

If one wire is violet, then no wire can be green.

Which of the following could be the complete set of wires in an acceptable cable?A green wire, a white wire and a violet wireA violet wire, a black wire and white wiresA red wire, a black wire and a green wireA yellow wire and exactly two black wiresIf exactly one black wire and exactly one white wire are used in an assembled cable, which of the following must be truethe cable contains no more than five wiresthe cable contains exactly six wiresthe cable contains a yellow wirethe cable does not contain a red wireThe maximum number of wires that can be used in an acceptable isa) 8 b) 7 c) 6 d) 4

If a white wire and a violet wire must be among the wires choose for a particular cable, any of the following pairs of wires could complete the cable except ablack wire and a second white wireyellow wire and second white wireyellow wire and a black wirered wire and a black wireIf an assembled cable consists of five wires, each a different colour, it could be true that a colour not used isa) Black b) White c) Green d) Red

If there is an additional requirement that violet wire must be used, if yellow is used, which of the following must be true?No cable contains fewer than six wiresNo cable contains more than five wiresGreen is never used if red is usedRed is always used if violet is usedDirections:

The manager of a radio program is going to future six Vocalists – M,N,P,Q,R and S – on an half-an-hour radio show during the course of one week. She will feature one vocalist on the show each day from Monday through Saturday. The manager must schedule the vocalists for the show according to the following conditions.

M must be featured earlier in the week than R

P must be featured on Tuesday

Q must be featured on the day immediately before or immediately after the day on which N is featured.

If N is to be featured on Thursday, the earliest day on which R can be featured isa) Monday b) Tuesday c) Wednesday d) Friday

If S is to be featured on Friday, M must be featured ona) Monday b) Tuesday c) Wednesday d) Thursday

If Q is to be featured on Thursday, the latest day on which M can be featured isa) Monday b) Tuesday c) Wednesday d) Friday

Which of the following vocalist can be featured on Monday?a) N b) P c) Q d) S

If S is to be featured on Thursday, which of the following is true?a) M must be featured on Wednesday b) N must be featured on Saturday c) Q must be featured exactly two days after R is featured

d) R must be featured on Wednesday

If S is to be featured on Friday, what is the total number of acceptable schedules available to the manager?a) 1 b) 2 c) 3 d) 4

Directions: For Question 28 to 30. Each Question or group of questions is based on a passage or set of conditions. In answering some of the questions, it may be useful to draw a rough diagram. For each question, select the best answer choice given.

Questions

An instrument has exactly six keys, each sounding a different note. The notes are K, L, M, N, O and P. Playing this instrument involves playing successive chords; each consisting of three of the notes produced s

simultaneously;

Notes K and N can only be produced simultaneously. Notes L cannot be produced simultaneously with note O.

Page 96: 7116273 Aptitude IGATE Infosys Tcs Ect

a chord can neither be the same as the one immediately is nor be the same as the one immediately after it.

Each of the following is an acceptable chord EXCEPTa) KLP b) KMN c) KNP d) LMP e) MOP

29.Chord LMP could be immediately followed by each of the following chords EXCEPT

a) KLN b) KMN c) KNP d) LMP e) MOP

Which of the following pairs of notes can be played simultaneously with K to produce an acceptable chord?a) LM b) LO c) LP d) MN e) OP

If both filling and emptying pipes of a pool are open, the pool fills in 10 hours. If just the filling pipes are open, the pool fills in 4 hours. How many hours does it take to empty the pool if just the emptying pipes are opena) more than 5 but less than 6 b) exactly 5 c) more than 6 d) exactly 6

a cube has an edge of 4, if the cube is split into cubes with edges of 2, what is the total surface area of the smaller cubes ?a) 32 b) 64 c) 96 d) 192

a man traveled along a straight road from a town P to town Q at a uniform rate of 30 miles/hour, he then returned at a uniform rate of 60 miles/hour. What was his average rate for the round trip in miles/hour?a) 35 b) 38 c) 40 d) 45

Exactly how many degrees are there between the hands of a clock at 3:40 pma) 110 b) 120 c) 130 d) 140

a man walked diagonally across a square plot. What was the percent saved by not walking along the edges?a) 20 b) 22 c) 24 d) 30

A gallon of oil is poured to a cubical container 7 inches on an edge. About how high in inches does the oil rise in the container? (I gallon = 231 cubic inches)a) 3.8 b) 4.2 c) 4.7 d) 5.2

the time allowed for completing a certain piece of work is 40 days. By extending the time by 10 days we find that 25 fewer man will be needed. How many men are required to finish the work in 40 days?a) 115 b) 125 c) 135 d) 145

A whistle blows every 14 minutes and a bell rings every 12 minutes. If the whistle and bell both sound together at 12 :00 noon, at what time, afternoon, will they first sound together again ?a) 2:48 b) 2:24 c) 1:54 d) 1:24

A truck can hold 3 cubic yards of sand. If a cubic foot of a sand weighs 80 pounds, the weight of the truck load of sand in pounds isa) 240 b) 270 c) 2160 d) 6480

A ship sails 60 miles south, 90 miles east and 60 miles south again. How many miles is it from its starting point ?a) 120 b) 135 c) 150 d) 160

41. The formula h= 16t2 represents the distance h; infeet, that an object falls from rest after 1 seconds. In how many seconds will an object fall 400 ft.

a) 2 b) 3 c) 4 d) 5

Technical

1.machine tool drives are generally given in:

a) arithmetic progression b)geometric progression

c) harmonic progression d)none of the above

2.finite element analysis is used in

a) accurate measurement b)stress analysis

c)metallurgy d)none of the above

3.coolant is generally not used while machining

a) low carbon steel b)high carbon steel

c) cast iron d)alloy steel

4.Machinability of steel is improved by the presence of

a)nickel b)tungsten

Page 97: 7116273 Aptitude IGATE Infosys Tcs Ect

c)sulphur d)chromium

5.negative rake angle

a)increases cutting force b)decreases cutting force

c)no effect on cutting force d)none of the above

6.the hardest tool material used is

a)coated carbide b)high speed steel

c)boron nitride d)diamond

7.stick slip phenomenon is associated with

a)spindle rotation b)belt drives

c)slide movement d)none of the above

8.Gear hobbing is

a)shaping process b)generating process

c)forming process

9.cutting speed of H.S.S tools while cutting low carbon steel

a)40m/min b)250m/min

c)500m/min d)none

10.cutting speed of carbide tools while cutting low carbon steel

a)40m/min b)250m/min

c)500m/min d)80m/min

11.straightness that can be achieved over a length of 300mm by grinding

a).005 -.01mm b)0.1-0.2mm

c)0.05-0.1mm d)0.5-0.8mm

12.the circularity that can be achieved by finish turning on a standard centre lathe is

a)0.005-0.008mm b)0.0-0.08mm

c)0.1-0.2mm d)0.2-0.3mm

13.honing can correct

a)oviality b)taper

c)straightness of bore d)axial curvature

14.Machine tool sliding surfaces are finish matched by

a)surface grinding b)fine milling

c)slide way grinding d)scraping

15.Close sliding fit is given by

a)H7/g6 b)H7/h6

c)H9/p6 d)H7/k6

16.60H6 can be achieved by

a)turning b)boring

c)drilling d)internal grinding

17.The displacement accuracy which can be achieved on a jig boring machine

a)5/µm b)50/µm c)100/µm

Page 98: 7116273 Aptitude IGATE Infosys Tcs Ect

18.Hardness value in cast iron can be measured in

a)shore b)BHN c)Rockwell d)none

19.Auto collimator is used to measure

a)dia upto 0.001mm b)lengths

c)straightness d)Involute error

20.Surface finish is measured by

a)comparator b)planograph

c)sprit level d)none

21.The outside diameter of a 20 teeth 3 module gear is

a)54 b)60 c)63 d)70

22.Herring bone gear is used for

a)High speed transmission b)noise free transmission

c)Better load transmission d)none

23.In a right angle transmission by helical gears, the hand of the helix of the gears are

a)same b)opposite c)Immaterial d)none

24.Worm drive is always self-locking

a)True b)false

25.Most hydrostatic bearings normally work under

a)constant flow system b)constant pressure system

c)constant temperature system d)constant volume system

26.Ball bearings are not generally used in crank shafts of automobiles because

a)of high cost b)of high speed c)cannot be assembled d)none

27.Bronze is used in journal bearings essential to

a)reduce friction b)reduce heat

c)eliminate oxidization d)eliminate seizing

28.The spindle of high precision cylindrical grinder is mounted on

a)Ball bearing b)Taper roller bearing

c)Hydrostatic bronze bearing d)needle bearing

29.To accelerate the flow of gases

a)nozzle is used b)diffuser is used

c)dia of pipe to increased d)none of these will help

30.Hydraulic accumulator

a)acts as an accumulator b)acts as a storage for hydraulic energy

c)acts as a pressure relief d)none

arrangement.

31.When ice which is floating in water, melts, does the level to water

a)go up b)go down c)remain the same.

32.Hammer blow in pipe happens due to

a)friction in pipes b)change in temperature

Page 99: 7116273 Aptitude IGATE Infosys Tcs Ect

c)stoppage of flow d)none

33.Temperature rise in fluid flow causes

a)reduction in viscosity b)increase in viscosity c)no effect

34.Ratio between velocity of sound and velocity of any object is

a)prandtl number b)Reynolds number

c)avagadro’s number d)mach number

35.Cavitaion effect in flow occurs due to

a)excess pressure b)excess temperature

c)excess flow d)none

36.In electrical locomotives ultimate transmission is by

a)AC Motor b)DC Shunt Motor

c)DC Series Motor d)Synchronous motor

37.Four beams of identical sections and lengths are made of cast iron, aluminium,steel and hardened steel. The beams are fixed at both ends; for a uniformly distributed load which will deflect least

a)cast iron b)aluminium c)steel d)hardened steel

38.Geneva mechanism is used for obtaining

a)Intermittent motion b)accelerating motion

c)continuous motion d)none

39.Corioles component is associated with

a)Displacement b)acceleration

c)velocity d)none

40.Effect of increase of outside diameter of a helical compression spring on its rigidity is

a)increased rigidity b)decreased rigidity

c)no effect d)dry friction

41.Jominey curve is related to the study of

a)hydraulics b)thermodynamics

c)electrical d)none

42.TTT diagram is used in

a)surface finish b)heat treatment

c)stress analysis d)thermodynamics

43.Mehanite is the name given to a type of

a)alloy steel b)granite

c)nodular cast iron d)cast iron

44.Unwanted impurity in molten aluminium is

a)oxygen b)carbon dioxide

c)nitrogen d)hydrogen

45.Hardness in steel is a s a result of

a)martensitic structure b)pearlitic structure

c)leduburetic structure d)austenitic structure

Page 100: 7116273 Aptitude IGATE Infosys Tcs Ect

46.Carburising is done

a)for blackening of high carbon steel b)for case hardening of

low carbon steel

c)for post hardening d)to increase machinablility

process to reduce of high tensile steels

internal stresses

47.piston of an IC engine is made out of

a)cast iron b)steel casting c)beryllium alloy d)aluminium alloy

48.The crank shaft of a 300 HP IC engine is

a)hot forged b)shell moulded

c)cold forged d)obtained through investment casting

A 6kg ball is suspended from two wires of equal length. The angle between the wires is 90 degrees. What is the tension in each wire?

58.8N41.6N29.4N83.2N

a 523 kg experimental rocked sted can be accelerated from rest to 1602 km/h in 1.82s. what net force is required?

4.6X106N4.6X105N1.6X106N1.3X105N

a force that averages 984N is applied to a 0.42kg steel ball moving at 13.8 m/s by a collision that lasts 0.027s. if the force is in a direction opposite to the initial velocity of the ball, what is the final speed of the ball?

77.1m/s63.3m/s13.8m/s49.5m/s

a stone initially at rest is dropped over the edge of a very tall building. Taking g as 10m/s2 , what was the change in the speed of the stone building during the 4th second?

10m/s30m/s15m/s40m/s

how long will it take a 2000w motor to lift a 320kg container vertically upwards for 26m?(g = 9.8m/s2)

29s6.3s4.2s4.1sa collision between two isolated items will in generally conservethe total kinetic energy and the total momentum of the systemthe total kinetic energy but not the total momentum of the systemthe total momentum but not the total kinetic energy of the systemthe momentum of each objectwhat is the force exerted on a submarine window of 2m2 area, if the submarine is at a depth of 2km below the surface?(the density of sea water is 1.03X103 kg/m3)

4.04X107N4.04X104N2.02X104N2.02X107N]

how many of the following statements are true:the buoyant force on a body depends on the shape of the bodythe buoyant force on a body always acts in the same directionthe buoyant force on a body depends on the mass of the body

Page 101: 7116273 Aptitude IGATE Infosys Tcs Ect

the buoyant force on a body depends on the density of the fluid surrounding the body

only oneonly twoonly threeonly four

how many of the following statements may be false?The pressure in a fluid depends on the density of the fluidThe forces due to pressure in a fluid act perpendicular to the surface of a body immersed in a fluidThe buoyant force experienced by a body immersed in a fluid depends on the density of the bodyThe buoyant force always acts vertically upwards on a body immersed in a fluid

Only oneOnly twoOnly threeAll four

A body partially floats with 2/3rds of its volume under the surface of a fluid. If the density of the fluid is X kg/m3, what is the density of the body?

3/2 X kg/m32/3 X kg/m314.7 X kg/m30.15 X kg/m3

how many atoms are present in a mole of CH4 ?(avagadro’s number = 6.023X1023 /mole)

1.2X10243.01X10241.2X10236.023X1023

a power station generates 500MW of power and exhausts 800MW as waste heat into the environment. What is the efficiency of the power station?

61.5%38.5%62.5%23%

how long does it take a 350W coffeepot to boil of 0.7L of water initially at 1000c. the latent heat of vapourisation of water = 2.1X106 J/kg.

6.1 min.7.0 min3.9 min.3.1 min.

a 16 ohm resistor and 8 ohm resistor are connected in series across a 12V battery. What is the voltage across each resistor respectively?

8V , 4V4V , 8V12V, 12V6V, 6V

how many of the following statements regarding sound are true?Intensity depends on the square root of the amplitudeLoudness only depends on the intensity of a soundThe pitch of a sound depends on the frequencySound is reflected whenever a change in acoustic impedance is encountered

Only oneOnly twoOnly threeAll four

How many of the following statements regarding light are true?Light travels in a straight line even when it crosses between two media of different refractive indexInfrared light has longer wavelength than ultra-violet lightLight can behave both like a ray and like a particleIt is possible to get a real image with a concave lense

Only oneOnly twoOnly threeAll four